You are on page 1of 80

VISION IAS ™ ...

Inspiring Innovation
www.visionias.in
www.visionias.wordpress.com

“The significant problems we face can not be solved at the same level of thinking we were at when we created them." - Albert
Einstein

ANALYSIS / APPROACH / SOURCE / STRATEGY: GENERAL STUDIES PRE 2016 PAPER


- TEAM VISION IAS

 The analysis is based on premise that UPSC wants to identify aspirants with certain skills and aptitude required to function as a good
administrator. UPSC does not explicitly describe these skills so we infer them from Syllabus, Question Paper and feedback of experts and
students.
 This analysis is an attempt to quantify the subjective understanding and make explicit the implicit.
 We have inferred some conclusions from these stats but you are free to infer differently. Moreover independent/impartial thinking is one
of the qualities expected in administrators.

Observations on CSP 2016

 The Paper portrayed a departure from the trend of last 4 years. Although, in 2015 the number of current affairs questions increased but in 2016
UPSC has drastically shifted their focus on Current Affairs.
 There were traditional questions from non-traditional sources eg. Question on Keshab Chandra Sen and Agrahattas.
 Paper was more factual compared to papers of CSAT era.
 In the current affairs section, instead of linking the current affairs questions to concepts it was more on factual side.
 Proportion of unapproachable questions has increased than previous year.
 Polity has moved towards traditional concepts compared to last year trend and is moving towards governance and public administration type
questions eg- Chief Secretary, Gram Nyayalayas.
1 www.visionias.in ©Vision IAS
 Economy questions were largely drawn from Current Affairs eg- . Ease of Doing Business, Amber Box, Blue Box Subsidies, MCLR, etc.
 Deceptive presentation is a common method to segregate deep knowledge with superficial knowledge or hunch. This technique was used in many
questions. i.e. some questions could be solved through eliminating one statement e.g. GIAHS, National Ganga River Basin Authority etc.
 Some questions were too easy (e.g. Pradhan Mantri Mudra Yojana, UDAY, etc). This does not mean that we should not have proper knowledge of
topic because some questions like Atal Pension Yojana required complete knowledge of the topic).
 Pointers for most of the questions could be traced to The Hindu but as recommended earlier, in the age of internet, complete knowledge of topic
is expected (Gram Nyayalayas, The Economics of Ecosystem and Biodiversity).

Recommendations:

 Read Basic/standard books, one Good News Paper (like The Hindu) and use internet extensively as almost 80% questions could be attempted with
this combination.
 Have keen perception about things going around you and assume less and less. After reading newspaper always clarify key terms from internet or
books or teachers.
 Read one good book on each traditional subjects this will help in prelims, mains and interview i.e. all stages of examination. This will also help in
making some unapproachable question approachable.
 This year with the inclusion of factual questions and state civil service/State Services type one liners, it is advisable to be ready for analytical and
factual questions at the same time. It is expected that UPSC will not ask irrelevant facts (Date on which someone died type asked in state civil
service) but important facts should not be ignored.
 Do not ignore your strong areas to do research in weak areas. You should be able to answer most of the questions from your strong areas.
 Nearly 60% of the questions were can be answered by reading elementary books and newspapers. Strategy should be to avoid negative marking in
tough questions and maximize your score by first finding the easy ones and completing them.
 Read questions carefully and use elimination technique wherever possible.

2 www.visionias.in ©Vision IAS


EM = Essential material like basic books etc.
RM = Reference material
EN = Essential News/Current Affairs
RR = Random Read like random website etc.
E : Easy , M : Medium , D : Difficult

Nature of Question
F: Fundamental, Conventional and conceptual question which is easily available in commonly recommended books. If a current affairs source is mentioned then it
indicate that you had one more reason to prepare this
FA: Fundamental Applied question is an analytical question which require information + application of mind.
Answer to these question are from commonly recommended books but not so obvious to find.
CA: Current affair question which can be answered almost completely using given source alone.
CAA(Current affair Applied): Current affair question which needs information from more than one source to answer AND/OR application of mind(common sense
and overall understanding) of aspirant. This is why source for these question may not be answering question completely.
FCA (Fundamental + Current affair): This is a F or FA question supplemented with current affairs or a current affairs question that needs background information for
complete answer. Source to these question might not answer question in entirety.
U (unconventional question): unconventional question means the question that is distantly related to syllabus provided by UPSC. It is neither present in even
reference sources(in addition to recommended books) nor prominently in news.

3 www.visionias.in ©Vision IAS


A
L
n Sou
e
s Natu rce
Q.N. Section Question v Source Explanation Motivation Vision IAS
w re Typ
e
e e
l
r
1 Current Regarding 'Atal Pension C M CAA http://pib.nic.in/ne EN Statements 1 and 3 are Financial Covered in PT 365 material -
Affairs Yojana', which of the wsite/PrintRelease. correct: To address the inclusion Social schemes.
following statements is/are aspx?relid=138244 longevity risks among the schemes like Asked in PT365 Test - Test 2102
correct? workers in unorganised sector APY, Consider the following statements
1. It is a minimum http://pib.nic.in/ne and to encourage the workers Suraksha with respect to Atal Pension
guaranteed pension scheme wsite/PrintRelease. in unorganised sector to Yojana,Jeev Yojana
mainly targeted at aspx?relid=116208 voluntarily save for their an Jyoti 1. Any Indian citizen having a
unorganized sector workers. retirement, APY was yojana, Jan savings bank account between the
2. Only one member of a launched. Dhan, etc. age of 18 to 45 years can subscribe
family can join the scheme. Statement 2 is not correct: have to this pension scheme.
3. Same amount of pension Atal Pension Yojana (APY) is remained in 2. There is a provision of voluntary
is guaranteed for the spouse open to all bank account news exit before the age of 60 years.
for life after subscriber's holders who are not members regularly. 3. This scheme will replace existing
death. of any statutory social security Swavalamban scheme.
Select the correct answer scheme. It doesn't bar the Which of statements given above
using the code given below. number of family members is/are not correct?
(a) 1 only from a family. (a) 1 only
(b) 2 and 3 only (b) 2 and 3 only
(c) 1 and 3 only (c) 1 and 2 only
(d) 1, 2 and 3 (d) 1, 2 and 3
2 Current The term 'Regional B E CA http://www.thehin EN Regional Comprehensive Trade Covered in both All India Test
Affairs Comprehensive Economic du.com/news/nati Economic Partnership (RCEP) negotiations serie s twice and PT 365 material
Partnership' often appears in onal/india-warned- Agreement among ASEAN + like TTP, Tests - 1722, 1770, 1895
the news in the' context of against-pitfalls-in- Six FTA Partners (Australia, TTIP, RCEP The 'Regional Comprehensive
the affairs of a group of asean-trade- China, India, Japan, South have Economic Partnership', recently in
countries known as agreement/article8 Korea and New Zealand): remained the news, is related to the affairs
(a) G2O 729070.ece Based on the Declaration of regularly in of:
(b) ASEAN the Leaders during the ASEAN news. (a) ASEAN
(c) SCO Summit in November 2012, (b) BRICS
(d) SAARC negotiations for a (c) OECD
comprehensive economic (d) SAARC
partnership between the 10 Test - 1723, 1774, 1891
ASEAN member states and its Regional Comprehensive Economic
six FTA partners commenced Partnership (RCEP) Agreement is
in May 2013. Ten rounds of proposed among ASEAN + six FTA

4 www.visionias.in ©Vision IAS


negotiations have so far been partners. Which among the
held. The 10th round was held following are members of RCEP?
from 8 to16 October 2015 in 1. New Zealand
Busan, Korea. 2. China
The negotiations cover a 3. Japan
number of areas like trade in 4. Papua New Guinea
goods, services, investment, 5. Russia
intellectual property, 6. India
economic and technical Select the correct answer using the
cooperation, competition, e- code given below.
commerce and legal and (a) 1, 2, 3 and 6 only
institutional issues. (b) 1, 3, 4 and 5 only
Papua New Guinea is not a (c) 2, 4 and 6 only
member of ASEAN. (d) 1, 2, 3, 4 and 6
3 Current On which of the following D D CAA http://www.thehin RR The Bureau initiated the Energy
Affairs can you find the Bureau of du.com/features/h Standards & Labeling efficiency
Energy Efficiency Star Label? omes-and- programme for equipment has
1. Ceiling fans gardens/conservin and appliances in 2006 to remained
2. Electric geysers g-power-holds-key- provide the consumer an continuously
3. Tubular fluorescent lamps to- informed choice about the in news.
Select the correct answer future/article8625 energy saving and thereby the
using the code given below. 361.ece cost saving potential of the
(a) 1 and 2 only relevant marketed product.
(b) 3 only http://pib.nic.in/ne The scheme is invoked for 21
(c) 2 and 3 only wsite/PrintRelease. equipment/appliances, i.e.
(d) 1, 2 and 3 aspx?relid=137917 Room Air Conditioners,
Tubular Fluorescent Tube
https://beeindia.go Lights, Frost Free
v.in/content/star- Refrigerators, Distribution
labelled-appliances Transformers, Induction
Motors, Direct Cool
Refrigerator, electric storage
type geyser, Ceiling fans,
Color TVs, Agricultural pump
sets, LPG stoves, Washing
machine, Laptops, ballast,
floor standing ACs, office
automation products, Diesel
Generating sets & Diesel
pumpsets.

5 www.visionias.in ©Vision IAS


4 Science and India is an important D E CAA http://www.thehin EN ITER (International In News Covered in PT 365 material
Technology member of the 'International du.com/opinion/bl Thermonuclear Experimental
Thermonuclear Experimental ogs/blogs-the- Reactor) is an international
Reactor'. If this experiment copernican/article nuclear fusion research and
succeeds, what is the 5685659.ece engineering megaproject,
immediate advantage for which will be the world's
India? largest magnetic confinement
(a) It can use thorium in plasma physics experiment.
place of uranium for power
generation
(b) It attain a global role in
satellite-navigation
(c) It can drastically improve
the efficiency of its fission
reactors in power generation
(d) It can build fusion
reactors for power
generation
5 Ancient In the context of the history D D FA The Penguin RR Taniyur, is a large village to test the
History of India, consider the History of Early under Chola administrative knowledge
following pairs: India: From the division. of ancient
Term Description Origins to AD 1300 Eripatti was a special category history in
1. Eripatti : Land, revenue - Romila Thapar of land known in South india. Depth
from which was set apart for This land was donated by
the maintenance of the individuals, revenue from
village tank which was set apart for the
2. Taniyurs : Villages donated maintenance of the village
to a single Brahmin or a tank.
group of Brahmins Land grants and generous
3. Ghatikas : Colleges donations were given to
generally attached to the educational institutions like
temples Ghatikas and Mathas. Mathas
Which of the pairs given were residential schools for
above correctly matched? early education in Pallava
(a) 1 and 2 kingdoms.
(b) 3 only
(c) 2 and 3
(d) 1 and 3

6 www.visionias.in ©Vision IAS


7 www.visionias.in ©Vision IAS
6 Current Consider the following A M CA http://www.thehin EN India and France have was Tests- 1722, 1770, 1895, 2060
Affairs statements: du.com/sci- launched an International constantly in With reference to International
1. The International Solar tech/energy-and- Solar Alliance to boost solar news. Solar Alliance (ISA), consider the
Alliance was launched at the environment/modi energy in developing following statements:
United Nations Climate -launches- countries. 1. It was launched by India and
Change Conference in 2015. international-solar- The initiative was launched at France.
2. The Alliance includes all alliance/article793 the UN Climate Change 2. It invites countries which are
the member countries of the 4560.ece Conference in Paris. It will be located between Tropic of Cancer
United Nations. a common platform for and Tropic of Capricorn.
Which of the statements cooperation among solar 3. It will work under International
given above is/are correct? resource rich countries lying Renewable Energy Agency (IRENA).
(a) 1 only fully or practically between Which of the statements given
(b) 2 only the Tropics of Cancer and above is/are correct?
(c) Both 1 and 2 Capricorn. (a) 1 only
(d) Neither 1 nor 2 It will not duplicate or (b) 1 and 2 only
replicate the efforts that (c) 2 and 3 only
others (like International (d) 1, 2 and 3
Renewable Energy Agency
(IRENA), Renewable Energy
and Energy Efficiency
Partnership (REEEP),
International Energy Agency
(IEA), Renewable Energy
Policy Network for the 21st
Century (REN21), United
Nations bodies, bilateral
organizations etc.) are
currently engaged in, but will
establish networks and
develop synergies with them.
7 Current 'European Stability B E CA http://www.thehin EN The European Stability was in news
Affairs Mechanism', sometimes du.com/news/inter Mechanism is a European constantly
seen in the' news, is an national/eu- Union agency that provides due to
(a) agency created by EU to begins-disbursal- financial assistance, in the Greece
deal with the impact of of-greece-bailout- form of loans, to eurozone bailout.
millions of refugees arriving money/article8756 countries or as new capital to
from Middle East 190.ece banks in difficulty.
(b) agency of EU that
provides financial assistance
to eurozone countries
(c) agency of EU to deal with

8 www.visionias.in ©Vision IAS


all the bilateral and
multilateral agreements on
trade
(d) agency of EU to deal with
the conflicts arising among
the member countries
8 Economy Which of the following is/are C M FA http://www.agriinf RR Advantages of Drip Irrigation: Due to “Per
the advantage/advantages of o.in/default.aspx?p 1. Maximum use of available Drop More
practicing drip irrigation? age=topic&superid water. Crop”
1. Reduction in weed =8&topicid=2243 2. No water being available to component
2. Reduction in soil salinity weeds. under
3. Reduction' in-soil erosion 3. Maximum crop yield. Pradhan
Select the correct answer 4. High efficiency in the use of Mantri
using the code given below. fertilizers. Krishi
(a) 1 and 2 only 5. Less weed growth and Sinchyee
(b) 3 only restricts population of Yojana
(c) 1 and 3 only potential hosts. (PMKSY),
(d) None of the above is an 6. Low labour and relatively Drip
advantage of practising drip low operation cost. irrigation
irrigation 7. No soil erosion. remained
8. Improved infiltration in soil regularly in
of low intake. news. Also
9. Ready adjustment to mentioned
sophisticated automatic in Eco
control. Survey
10. No runoff of fertilizers into
ground water.
11. Less evaporation losses of
water as compared to surface
irrigation.
12. Improves seed
germination.
13. Decreased to tillage
operations.
Disadvantages of Drip
Irrigation:
In spite of the fact that drip
irrigation has so many
potential benefits , they’re a
certain limitation also, there
are as follow:

9 www.visionias.in ©Vision IAS


1. Sensitivity to clogging
2. Moisture distribution
problem 3. Salinity hazards
4. High cost compared to
furrow.
5. High skill is required for
design, install and operation.
9 Current Regarding 'DigiLocker', C E CA http://pib.nic.in/ne EN Digital Locker is one of the key Have Test – 2084 (PT365) Which of the
Affairs sometimes seen in the news, wsite/PrintRelease. initiatives under the Digital remained in following statements regarding
which of the following aspx?relid=142414 India Programme. news digital locker under Digital India
statements is/are correct? Digital Locker is aimed at regularly. program is/are correct?
1. It is a digital locker system http://www.thehin minimising the usage of 1. It serves as a Uniform Resource
offered by the Government du.com/todays- physical documents and Identifier of e-documents issued
under Digital India paper/tp- enable sharing of e- by Government agencies.
Programme. national/sc- documents across agencies. 2. Users can upload their e-
2. It allows you to access dismisses-pil-on- documents only with digital
your e-documents digital- signatures.
irrespective of your physical locker/article7700 3. It is mandatory to link
location. 119.ece individual's digital locker with
Select the correct answer Aadhar number to upload
using the code given below. documents.
(a) 1 only Select the correct answer using the
(b) 2 only code given below.
(c) Both 1 and 2 (a) 1 only (b) 1 and 3 only
(d) Neither 1 nor 2 (c) 2 and 3 only (d) 1, 2 and 3
10 Current Recently, linking of which of B E CA http://www.thehin EN The plan is to eventually Remained Asked in All India Test series
Affairs the following rivers was du.com/news/nati divert water from the continuously 1718, 1769, 2066
undertaken? onal/andhra- Polavaram dam, since the in news. The Polavaram Reservoir being
(a) Cauvery and Tungabhadra pradesh/interlinkin dam is still under construction built to interlink Godavari and
(b) Godavari and Krishna g-godavari-krishna- and will take at least 4 to 5 Krishna extends to which of the
(c) Mahanadi and Son my-most- years to be ready, the following states?
(d) Narmada and Tapti satisfying- Chandrababu Naidu 1. Andhra Pradesh
exercise/article805 government decided to divert 2. Telangana
1320.ece Godavari water from the 3. Odisha
Pattiseema Lift Irrigation 4. Chhattisgarh
http://indianexpre Scheme. This plan kicked off Select the correct answer using the
ss.com/article/expl on September 16, 2015. code given below.
ained/godavari- (a) 1 and 2 only (b) 1, 2 and 3 only
and-krishna-rivers- (c) 1, 2 and 4 only (d) 1, 2, 3 and 4
interlink-when-
two-rivers-meet/

10 www.visionias.in ©Vision IAS


11 Environment In the cities of our country, B M FCA http://www.thehin EN National Air Quality Index: Important Asked in All India Test series
which among the following du.com/opinion/e There are six AQI categories, initiative of Tests - 1725, 1776
atmospheric gases are ditorial/air- namely Good, Satisfactory, Minister for Which of the following air
normally considered in pollution-in-india- Moderately polluted, Poor, Environmen pollutants is/are included in the
calculating the value of Air clean-air-agenda- Very Poor, and Severe. t, Forests & National Air Quality Index?
Quality Index? for-the- The AQI considers eight Climate 1. Sulphur dioxide
1. Carbon dioxide cities/article82725 pollutants (PM10, PM2.5, Change. 2. Ozone
2. Carbon monoxide 73.ece NO2, SO2, CO, O3, NH3, and Also 3. Carbon dioxide
3. Nitrogen dioxide Pb) for which short-term (up remained in 4. Lead
4. Sulfur dioxide http://pib.nic.in/ne to 24-hourly averaging period) news due to 5. Ammonia
5. Methane wsite/PrintRelease. National Ambient Air Quality high Select the correct answer using the
Select the correct answer aspx?relid=110654 Standards are prescribed. Particulate code given below.
using the code given below. matters in (a) 1, 2 and 3 only
(a) 1, 2 and 3 only many cities. (b) 1, 2, 4 and 5 only
(b) 2, 3 and 4 only (c) 2, 3 and 5 only
(c) 1, 4 and 5 only (d) 1, 3 and 4 only
(d) 1, 2, 3, 4 and 5
12 Science and With reference to 'Astrosat',' D D CA http://www.thehin EN ASTROSAT is India’s first was Asked twice in All India Test series
Technology the astronomical du.com/sci- dedicated multi wavelength constantly in Test -
observatory launched by tech/science/isro- space observatory. ASTROSAT news 1719,1767,2058,2061,2064,2067
India, which of the following launches-astrosat- will observe universe in the With reference to ASTROSAT
statements is/are correct? first-space- optical, Ultraviolet, low and mission, consider the following
1. Other than USA and observatory/article high energy X-ray regions of statements:
Russia, India is the only 7697707.ece the electromagnetic 1. It is India’s first dedicated space
country to have launched a spectrum. India’s observatory observatory.
similar observatory into http://www.isro.go will be the fourth in space, 2. It will enable simultaneous
space. v.in/Spacecraft/ast after the Hubble(USA), multi-wavelength observations
2. Astrosat is a 2000 kg rosat Russia’s Spektr R and Suzaku with a single satellite.
satellite placed in an orbit at of Japan. 3. It is placed in geosynchronous
1650 km above the surface ASTROSAT with a lift-off mass orbit to make communication
of the Earth. of about 1513 kg was effective on earth.
Select the correct answer launched into a 650 km orbit Which of the statements given
using the code given below. inclined at an angle of 6 deg above is/are correct?
(a) 1 only to the equator by PSLV-C30. (a) 1 only
(b) 2 only ASTROSAT will observe (b) 1 and 2 only
(c) Both 1 and 2 universe in the optical, (c) 2 and 3 only
(d) Neither 1 nor 2 Ultraviolet, low and high (d) 1, 2 and 3
energy X-ray regions of the Tests - 1778. 1896
electromagnetic spectrum, Which of the following is/are space
whereas most other scientific observatory?
satellites are capable of 1. Astrosat

11 www.visionias.in ©Vision IAS


observing a narrow range of 2. Hubble
wavelength band. 3. Spektr
4. Suzaku
Select the correct answer from
code given below.
(a) 1 only
(b) 1 and 2 only
(c) 2 and 3 only
(d) 1, 2, 3 and 4
13 Medieval With reference to the C D F Early Medieval RR The 'ara-ghatta' originates Knowledge
History economic history of Indian Society - from the blend of the words of History in
medieval India, the term R.S. Sharma "ara" which means talked and Depth
'Araghatta' refers to "ghatta" which means pot.
(a) bonded labour A History of There is confirmation to
(b) land grant made to Ancient and Early contend that this
military officers Medieval India - arrangement of lifting water
(c) waterwheel used in the Upinder Singh from open wells was
irrigation of land presumably designed in the
(d) wasteland converted to India of the past.
cultivated land
14 Ancient With reference to the D D F Ancient Indian RR During Gupta period, the Knowledge
History cultural history of India, the Social History: memorizing of chronicles, of History in
memorizing of chronicles, Some dynastic histories or epic tales Depth
dynastic histories and epic Interpretations - was the work of a different
tales was the profession of Romila Thapar group of people, the sutas
who of the following? and magadhas.
(a) Shramana
(b) Parivraajaka
(c) Agrahaarika
(d) Maagadha
15 Current Recently, for the first time in D M CA http://www.thehin EN The Blue Mormon is a large, was in news Asked in All India Test series -
Affairs our country, which of the du.com/news/nati swallowtail butterfly found 1716
following States has declared onal/other- primarily in Sri Lanka and Consider the following statements
a particular butterfly as states/maharashtr India, mainly restricted to the about Blue Mormon, a species of
‘State Butterfly’? a-gets-state- Western Ghats of Butterfly, which was recently in
(a) Arunachal Pradesh, butterfly/article73 Maharashtra, South India and news:
(b) Himachal Pradesh 42955.ece coastal belts. It may 1. It is endemic to India.
(c) Karnataka occasionally be spotted in the 2. It is listed as an endangered
(d) Maharashtra Maharashtrian mainland species by International Union for
between Vidarbha and Conservation of Nature (IUCN)
Western Maharashtra. 3. It has been declared as 'State
12 www.visionias.in ©Vision IAS
It is reportedly the second butterfly' of Karnataka.
largest butterfly found in Which of the statements given
India, just smaller than the above is/are not correct?
southern birdwing. (a) 1 only
Not uncommon. Not thought (b) 1 and 2 only
to be threatened. Occurs (c) 2 and 3 only
throughout the year but more (d) 1, 2 and 3
common in the monsoon and
immediately after it.
Maharashtra has become the
first State in the country to
have a ‘State butterfly.’ Thus
all statements are wrong.
16 Science and Consider the following C M CAA http://www.thehin EN Statements 1 and 3 are It has Covered in PT 365
Technology statements: The Mangalyaan du.com/sci- correct. remained
launched by ISRO tech/science/articl The Mars Orbiter Mission regularly in
1. is also called the Mars e6434096.ece (MOM) has various news
Orbiter Mission achievements to its credit
2. made India the second http://pib.nic.in/ne after it was inserted into Mars
country to have a spacecraft wsite/PrintRelease. orbit on September 24, 2014.
orbit the Mars after USA aspx?relid=117336 The details of significant
3. made India the only achievements of MOM
country to be successful in http://isro.gov.in/p include
making its spacecraft orbit slv-c25-mars- i. First interplanetary mission
the Mars in its very first orbiter- realized by India and first
attempt mission/mars- Indian spacecraft to
Which of the statements orbiter-mission- incorporate full scale on-
given above is/are correct? profile board autonomy to overcome
(a) 1 only the long distances and the
(b) 2 and 3 only communication gaps due to
(c) 1 and 3 only non-visibility periods.
(d) 1, 2 and 3 ii. First Indian spacecraft to
successfully survive Van Allen
belt crossing 39 times.
iii. First Indian spacecraft to
escape the Sphere Of
Influence of Earth and orbit
Sun.
iv. First Mars mission in the
world to succeed Mars Orbit
Insertion in first attempt.

13 www.visionias.in ©Vision IAS


v. Most economical
interplanetary mission in the
world and paved way for cost-
effective access to deep
space. The launch vehicle,
Spacecraft and Ground
Segment have been realised
with a budget of Rs 450 Cr.
Statement 2 is not correct.
Since the first successful flyby
in 1965, four entities have
successfully made it to Mars:
NASA, the Soviet Union, the
European Space Agency and
the Indian Space Research
Organization, while others,
including Japan and China,
have tried.
17 Modern What was the main reason B E F NCERT, Bipin EM Surat Split refers to the Basic
History for the split in the Indian Chandra, Spectrum splitting of the Congress party question of
National Congress at Surat in into 'Moderates' and modern
1907? 'Extremists' after a clash at indian
(a) Introduction of the session. The extremists history
communalism into Indian were led by Lokmanya Tilak,
politics b Lord Minto Lajpat Rai and Sri Aurobindo
(b) Extremists’ lack of faith in and the moderates were led
the capacity of the by Gopal Krishna Gokhale,
moderates to negotiate with Pherozeshah Mehta and
the British Government Surendranath Banerjee. The
(c) Foundation of Muslim divided Congress re-united in
League the crucial Lucknow session of
(d) Aurobindo Ghosh’s congress in 1916.
inability to the elected as the
President of the Indian
National Congress
18 Modern The plan of Sir Stafford D E F NCERT, Bipin EM Cripps Mission was sent to basic Asked in All India Test series
History Cripps envisaged that after Chandra, Spectrum India under the president ship question of Tests-
the Second World War of Stafford Cripps in march modern 1721,1772,1894,2062
(a) India should be granted 1942. The intention was to get indian With reference to dominion status,
complete independence Indian support for Britishers in history consider the following statements:
(b) India should be the ongoing World War II. It 1. Nehru Report demanded

14 www.visionias.in ©Vision IAS


partitioned into two before proposed: dominion status.
granting independence An Indian Dominion that will 2. Cripps Mission proposed
(c) India should be made a be free to decide its relations dominion status.
republic with the condition with the commonwealth and 3. India became a dominion before
that she will join the United Nations gaining independence.
Commonwealth A constituent assembly to be Which of the statements given
(d) India should be given convened to frame a new above is/are correct?
Dominion status constitution. (a) 1 and 2 only
Any province not willing to (b) 1 and 3 only
join the Union could have a (c) 2 and 3 only
separate constitution and (d) 1, 2 and 3
form a separate.
Other points in the proposal
were- Acceptance of Indian
constitution by British
government would be subject
to condition that any province
would have the right to have
separate union and a treaty to
be negotiated to effect the
power transfer to safeguard
racial and religious minorities
Defense of India to remain in
British hands.
19 Geography Consider the following pairs: C D FA ATLAS, NCERT EM Bagelkhand or Baghelkhand is All the
Famous Place Region Geography books a region and also a mountain places are
1. Bodhgaya : Baghelkhand range in central India that famous for
2. Khajuraho : Bundelkhand covers the northeastern pilgrimages
3. Shirdi : Vidarbha regions of Madhya Pradesh and to
4. Nasik (Nashik) : Malwa and a small area of western promote
5. Tirupati : Rayalaseema Uttar Pradesh. Bodhgaya is in tourism
Which of the pairs given Bihar. government
above are correctly The Khajuraho group of has recently
matched? monuments was built during focused on
(a) 1, 2 and 4 the rule of the Rajput developing
(b) 2, 3, 4 and 5 Chandela dynasty. The religious
(c) 2 and 5 only building activity started circuits.
(d) 1, 3, 4 and 5 almost immediately after the
rise of their power,
throughout their kingdom to
be later known as
Bundelkhand.
15 www.visionias.in ©Vision IAS
Vidarbha is the eastern region
of the Indian state of
Maharashtra, comprising
Nagpur Division and Amravati
Division. Shirdi is a town and
falls under Ahmednagar District
in Maharashtra. It is not in
Vidarbha region.
Tirupati lies in the Rayalaseema
region of Andhra Pradesh.
20 Polity and The Parliament of India D E F Laxmikanth EM If Rajya Sabha passes a Basic
Governance acquires the power to resolution by a majority of not question of
legislate on any item in the less than two-thirds of Polity
State List in the national members present and voting
interest if a resolution to saying that it is “necessary or
that effect is passed by the expedient in the national
(a) Lok Sabha by a simple interest” that Parliament
majority of its total should make a law on a
membership matter enumerated in the
(b) Lok Sabha by a majority State List, Parliament
of not less than two-thirds of becomes empowered to make
its total membership a law on the subject specified
(c) Rajya Sabha by a simple in the resolution, for the
majority of its total whole or any part of the
membership territory of India. Such a
(d) Rajya Sabha by a majority resolution remains in force for
of not less than two-thirds of a maximum period of one
its members present and year but this period can be
voting extended by one year at a
time by passing a similar
resolution further.
21 Current Recently, which of the D D CA http://pib.nic.in/ne EN Government of Rajasthan is
Affairs following States has explored wsite/PrintRelease. planning to develop Inland
the possibility of aspx?relid=125015 Shipping Port at Jalore which
constructing an artificial would be connected to the
inland port to be connected Arabian Sea by developing a
to sea by a long navigational channel along the Kutch
channel? Creek.
(a) Andhra Pradesh
(b) Chhattisgarh
(c) Karnataka (d) Rajasthan

16 www.visionias.in ©Vision IAS


17 www.visionias.in ©Vision IAS
22 Environment With reference to the B M CAA http://www.thehin EN 21st Conference of Parties was Tests - 1779, 1784, 2070, 2087
Agreement at the UNFCCC du.com/news/inter (COP21) also known as Paris constantly in (PT365)
Meeting in Paris in 2015, national/cop-21- Agreement successfully news With reference to the Paris
which of the following historic-paris- concluded in Paris under the Agreement, consider the following
statements is/are correct? climate-pact- United Nations Framework statements:
1. The Agreement was signed adopted/article798 Convention on Climate 1. It was convened under the Inter
by all the member countries 1450.ece Change (UNFCCC). The Governmental Panel on Climate
of the UN and it will go into agreement is scheduled to go Change (IPCC).
effect in 2017. http://pib.nic.in/ne into effect from 2020. 2. This Universal Agreement will
2. The Agreement aims to wsite/PrintRelease. (Statement 1 is not correct). succeed the Nagoya Protocol.
limit the greenhouse gas aspx?relid=138511 One of the main focus of the Which of the statements given
emissions so that the rise in agreement is to hold the above is/are correct?
average global temperature increase in the global average (a) 1 only
by the end of this century temperature to well below (b) 2 only
o
does not exceed 2 C or even 2°C above pre- industrial level (c) Both 1 and 2
0
1.5 C above pre-industrial and on driving efforts to limit (d) Neither 1 nor 2
levels. it even further to 1.5°C. It Which of the following statements
3. Developed countries covers all the crucial areas regarding Green Climate Funds
acknowledged their identified as essential for a is/are correct?
historical responsibility in comprehensive and balanced 1. It was formally established by a
global warming and agreement, including UNFCCC decision in Paris at the
committed to donate $ 1000 mitigation, adaptation, loss COP 21.
billion a year from 2020 to and damage, finance, 2. It is meant to limit or reduce
help developing countries to technology development and greenhouse gas emissions in both
cope with climate change. transfer, capacity building and developed and developing
Select the correct answer transparency of action and countries.
using the code given below. support. A marked departure 3. It aims to jointly mobilize USD
(a) 1 and 3 only from the past is the 100 billion per year by 2020
(b) 2 only Agreement’s bottom-up Select the correct answer using the
(c) 2 and 3 only approach, allowing each code given below.
(d) 1, 2 and 3 nation to submit its own (a) 1 only
national plan for reducing (b) 3 only
greenhouse gas emissions, (c) 2, 3 only
rather than trying to repeat a (d) 1, 2 and 3 only
top-down approach
advocated by the Kyoto
Protocol, giving each country
an emission reduction target.
Statement 3 is not correct.

18 www.visionias.in ©Vision IAS


23 Environment Consider the following B M CAA http://www.thehin EN Statement 1 is not correct: was Asked twice in Test series:
statements: du.com/news/inter The ‘Club of Rome’ is a global constantly in Test: 1713,2060
1. The Sustainable national/un- think tank is a global think news Which of the followings are the
Development Goals were officially-adopts- tank which in its 1972 book differences between Millennium
first proposed in 1972 by a ambitious-new- “Limits to Growth” talked Development Goals (MDG) and
global think tank called the set-of-global- about sustainability; however Sustainable Development Goals
‘Club of Rome’. goals/article76897 they didn't propose (SDG)?
2. The Sustainable 26.ece Sustainable Development 1. MDG were in effect from 2000
Development Goals have to Goals. As per the resolution of to 2015 while SDG will remain in
be achieved by 2030. http://pib.nic.in/ne UN General Assembly in effect from 2015 to 2030.
Which of the statements wsite/PrintRelease. September, 2015 the 2. MDG consisted of 18 targets,
given above is/are correct? aspx?relid=136880 Sustainable Development while SDG consists of 169 targets.
(a) 1 only Goals (SDGs) have been 3. There are specific targets in
(b) 2 only adopted by 193 member SDGs called “Urban Goals” which
(c) Both 1 and 2 nations. were not included under the
(d) Neither 1 nor 2 Statement 2 is correct: These MDGs.
developmental schemes are Select the correct answer using the
mostly addressing the code given below.
Sustainable Development (a) 1 and 2 only
Goals (SDGs) which are (b) 1 and 3 only
targeted to be achieved by (c) 2 and 3 only
2030. (d) 1, 2 and 3
Test: 1719,1767,2058,2061,2067
Which of the following statements
is/are correct about New
Sustainable Development Agenda
(SDG) adopted by United Nations?
1. There are 169 Sustainable
Development Goal as part of the
Agenda.
2. The goals are non-binding to the
participatory nations.
3. It covers climate related targets
which are covered under UNFCC.
Select the correct answer using the
code given below.
(a) 1 only
(b) 2 only
(c) 2 and 3 only
(d) 1 and 3 only

19 www.visionias.in ©Vision IAS


24 Current A recent movie titled The A M CA http://www.thehin EN ‘The Man Who Knew Infinity’, In news
Affairs Man Who Knew Infinity is du.com/features/ci movie is based on a book by
based on the biography of nema/cinema- the same name written by
(a) S. Ramanujan reviews/the-man- Robert Kanigel on the life and
(b) S. Chandhrashekhar who-knew-infinity- times of Ramanujan. The 46th
(c) S. N. Bose review-a- IFFI opened with the film
(d) C. V. Raman misunderstood-
mind/article85345
74.ece

http://www.pib.nic
.in/newsite/mbErel
.aspx?relid=131773
25 Polity and Consider the following B M FA LAXMIKANTH EM The minimum age for Basic Asked in All India Test series Tests
Governance statements: contesting elections to question of - 1714, 1763 , 1765
1. The minimum age panchayats is 21 years. Polity With reference to elections to
prescribed for any person to In case of dissolution of Panchayats, consider the following
be a member of Panchayat is Panchayats before expiry of statements:
25 years. its term, it is mandatory to 1. The minimum age for contesting
2. A Panchayat reconstituted hold elections within 6 elections to panchayats is 25 years.
after premature dessolution months of its dissolution. 2. Elections to panchayats are
continues only for the Every Panchayat shall conducted as per the electoral
remainder period. continue for full term of 5 rolls prepared for the most recent
Which of the statements years. However, a Panchayat state election.
given above is/are correct? reconstituted after premature 3. The chairpersons of panchayats
(a) 1 only dissolution shall continue for at all three levels must be elected
(b) 2 only only the reminder of the indirectly by and from amongst the
(c) Both 1 and 2 period. If the period is less elected members.
(d) Neither 1 nor 2 than 6 months then it is not Which of the statements given
mandatory to held the above is/are not correct?
elections. (a) 1 only
(b) 1 and 3 only
(c) 2 and 3 only
(d) 1, 2 and 3
With reference to Panchayati Raj
Institutions (PRIs), consider the
following statements:
1. In case of dissolution of
panchayats before expiry of its
term, it is mandatory to hold
elections within 6 months of its

20 www.visionias.in ©Vision IAS


dissolution.
2. A panchayat reconstituted after
premature dissolution shall
continue for full 5 years.
3. All persons who have attained
minimum 25 years are eligible for
elections in panchayats.
Which of the statements given
above is/are correct?
(a) 1 only
(b) 1 and 3 only
(c) 2 and 3 only
(d) None
26 Polity and Which of the following B M FA LAXMIKANTH EM When the Lok Sabha is basic Asked in Test- 1785
Governance statements is/are correct? dissolved, all business concept of Which of the following bills lapse
1. A Bill pending in the Lok including bills, motions, polity after dissolution of Lok Sabha?
Sabha lapses on its resolutions, notices, petitions 1. A bill introduced in the Lok
prorogation. and so on pending before it or Sabha and pending in Lok Sabha.
2. A Bill pending in the Rajya its committees lapse. They (to 2. A bill passed by the Lok Sabha
Sabha, which was not been be pursued further) must be but pending in the Rajya Sabha.
passed by the Lok Sabha, reintroduced in the newly- 3. A bill pending in the Rajya Sabha
shall not lapse on dosslution constituted Lok Sabha. but not passed by the Lok Sabha.
of the Lok Sabha. However, some pending bills Select the correct answer using the
Select the correct answer and all pending assurances code given below.
using the code given below. that are to be examined by (a) 1 and 2 only
(a) 1 only the Committee on (b) 1 and 3 only
(b) 2 only Government Assurances do (c) 2 and 3 only
(c) Both 1 and 2 not lapse on the dissolution of (d) 1, 2 and 3
(d) Neither 1 nor 2 the Lok Sabha.
The position with respect to
lapsing of bills is as follows:
A bill pending in the Lok
Sabha lapses (whether
originating in the Lok Sabha or
transmitted to it by the Rajya
Sabha).
A bill passed by the Lok Sabha
but pending in the Rajya
Sabha lapses.
A bill not passed by the two
Houses due to disagreement

21 www.visionias.in ©Vision IAS


and if the president has
notified the holding of a joint
sitting before the dissolution
of Lok Sabha, does not lapse.
A bill pending in the Rajya
Sabha but not passed by the
Lok Sabha does not lapse.
A bill passed by both Houses
but pending assent of the
president does not lapse.
A bill passed by both Houses
but returned by the president
for reconsideration of Houses
does not lapse.
27 Current Which of the following is/are C M CAA http://www.thehin EN The Global Hunger Index (GHI) India ranks Test – 2086 (PT365)
Affairs the indicator/ indicators du.com/news/nati is designed to improved in Which of the following statements
used by IFPRI to compute the onal/india-is- comprehensively measure Global regarding the Global Hunger Index
Global Hunger Index Report? home-to-194- and track hunger globally and hunger is/are correct?
1. Undernourishment million-hungry- by country and region. index 2015 1. It is prepared annually by the
2. Child stunting people- Calculated each year by the to 55 from Food and Agricultural
3. Child mortality un/article7255937. International Food Policy 63 Organization.
Select the correct answer ece Research Institute (IFPRI), the 2. It gives equal weightage to child
using the code given below. GHI highlights successes and undernourishment, undernutrition
(a) 1 only https://www.ifpri. failures in hunger reduction and mortality.
(b) 2 and 3 only org/topic/global- and provides insights into the 3. As per the latest GHI report
(c) 1, 2 and 3 hunger-index drivers of hunger. By raising India has moved up to 55th
(d) 1 and 3 only awareness and understanding position from 63 which shows
of regional and country increasing hunger population.
differences in hunger, the GHI Select the correct answer using the
aims to trigger actions to code given below.
reduce hunger. (a) 1 only
To reflect the (b) 2 only
multidimensional nature of (c) 1, 3 only
hunger, the GHI combines the (d) 1, 2 and 3
following four component
indicators into one index:
Undernourishment: the
proportion of undernourished
people as a percentage of the
population (reflecting the
share of the population

22 www.visionias.in ©Vision IAS


whose caloric intake is
insufficient;
Child wasting: the proportion
of children under the age of
five who suffer from wasting
(that is, low weight for their
height, reflecting acute
undernutrition);
Child stunting: the proportion
of children under the age of
five who suffer from stunting
(that is, low height for their
age, reflecting chronic
undernutrition); and
Child mortality: the mortality
rate of children under the age
of five (partially reflecting the
fatal synergy of inadequate
nutrition and unhealthy
environments).
28 Economy There has been a persistent C M FA UPSC 2015 EM Budget deficit is used to Current
deficit budget year after previous question define a status of financial economic
year. Which action/actions of http://www.thehin health in which expenditures situation
the following can be taken by du.com/business/E exceed revenue. Reducing the and problem
the Government to reduce conomy/subsidies- revenue expenditure will of growing
the deficit? to-be-rationalised- certainly help in bridging the fiscal deficit.
1. Reducing revenue joint-session-on- gap. Subsidies are a part of It is also a
expenditure insurance-arun- expenditure and rationalizing repeated
2. Introducing new welfare jaitley/article6811 them would reduce the question
schemes 966.ece deficit. from last
3. Rationalizing subsidies Introducing new welfare year.
4. Reducing import duty schemes will most likely result
Select the correct answer in increasing expenditure and
using the code given below. so will expanding industries
(a) 1 only (b) 2 and 3 only which would require capital
(c) 1 and 3 only infusion.
(d) 1, 2, 3 and 4
29 Economy The establishment of B M FCA http://www.thehin EN In August 2015, Reserve Bank Last year RBI Covered in Vision Ias PT
‘Payment Banks’ is being du.com/business/a of India granted 'in-principle' granted ‘in 365(Economy)
allowed in India to promote ll-you-need-to- approval to 11 applicants to principle’ Tests -1718, 1766, 1890, 2060
financial inclusion. Which of know-about- start payments banks. These approval for With reference to payment banks,

23 www.visionias.in ©Vision IAS


the following statements payment- include Reliance Industries, payment consider the following statements:
is/are correct in this context? banks/article75613 Airtel M Commerce Services, banks to 11 1. Payment banks may take
1. Mobile telephone 53.ece Tech Mahindra, Vodafone m- entities, deposits and remittances, internet
companies and supermarket pesa, Aditya Birla Nuvo, including big banking and other specified
chains that are owned and https://rbi.org.in/s Department of Posts, names like services but they cannot
controlled by residents are cripts/BS_PressRel Cholamandalam Distribution Reliance undertake lending services.
eligible to be promoters of easeDisplay.aspx?p Services, Fino PayTech, Industries, 2. They can issue credit cards but
Payment Banks. rid=32615 PayTm, National Securities Aditya Birla not debit cards.
2. Payment Banks can issue Depository Ltd (NSDL) and Nuvo and 3. An FDI of 100% is allowed in
both credit cards and debit Sun Pharma. They can issue Tech these banks.
cards. debit cards but not credit Mahindra, 4. The Department of Posts has
3. Payment Banks cannot cards.FDI of 74% is allowed in as also Airtel been granted 'in-principle'
undertake lending activities. these banks. and approval by the RBI to start such
Select the correct answer Vodafone. banks.
using the code given below. Which of the statements given
(a) 1 and 2 only above is/are correct?
(b) 1 and 3 only (a) 1 and 2 only (b) 1 and 4 only
(c) 2 only (c) 1, 3 and 4 only (d) 1, 2, 3 and 4
(d) 1, 2 and 3
30 Science and With reference to ‘LiFi’, C M CAA http://www.thehin EN Using light to deliver wireless Path- Covered in PT365 Asked in All
Technology recently in the news, which du.com/todays- internet will also allow breaking India Test series -
of the following statements paper/tp- connectivity in environments technology 1721,1769,1895,2059,2067
is/are correct? international/lifi- that do not currently readily of Li-Fi uses What is the difference between Li-
1. It uses light as the medium data-transmission- support Wi-Fi, such as aircraft light to carry Fi and Wi-Fi technology?
for high-speed data through- cabins, hospitals and data and it 1. Li-Fi uses visible light while Wi-Fi
transmission. light/article621889 hazardous environments. draws lot of runs on radio waves.
2. It is a wireless technology 2.ece Light is already used for data attention all 2. Li-Fi is easier to hack compared
and is several times faster transmission in fibre-optic over the with Wi-Fi
than ‘WiFi’. cables and for point to point world. 3. Li-Fi transmits data at a higher
Select the correct answer links, but Li-Fi is a special and speed as compared to Wi-Fi.
using the code given below. novel combination of Which of the statements given
(a) 1 only technologies that allow it to above is/are correct?
(b) 2 only be universally adopted for (a) 1 only
(c) Both 1 and 2 mobile ultra high speed (b) 1 and 3 only
(d) Neither 1 nor 2 internet communications. (c) 2 and 3 only
Using the visible light (d) 1, 2 and 3
spectrum, Li-Fi technology can
transmit data and unlock
capacity which is 10,000 times
greater than that available
within the radio spectrum.

24 www.visionias.in ©Vision IAS


31 Environment The term ‘Intended B E CAA http://www.thehin EN Intended Nationally Recently Covered in PT365(Environment)
Nationally Determined du.com/news/nati Determined Contributions India along Asked in All India Test series Test
Contributions’ is sometimes onal/india-sets- (INDCs) is a term used under with other 1725, 1776
seen in the news in the ambitious-goals-to- the United Nations countries Which of the following are
context of tackle-climate- Framework Convention on submitted objectives of India's Intended
(a) pledges made by the change/article7715 Climate Change (UNFCCC) for its its Nationally Determined
European countries to 679.ece reductions in greenhouse gas Intended Contributions (INDCs)?
rehabilitate refugees from emissions that all countries Nationally 1. Increase the forest cover to
the war-affected Middle East Economic survey that signed the UNFCCC were Determined 33%.
(b) plan of action outlined by asked to publish in the lead up Contribution 2. Install 175 GW of solar, wind
the countries of the world to http://unfccc.int/f to the 2015 United Nations s (INDCs) to and biomass electricity by 2022.
combat climate change ocus/indc_portal/it Climate Change Conference the UN 3. Reducing emission intensity by
(c) capital contributed by the ems/8766.php held in Paris, France in Framework 1/3rd by 2030.
member countries in the December 2015. Convention 4. create additional carbon sink of
establishment of Asian http://indianexpre The INDCs, which lay out the on Climate around 3 billion by 2030.
Infrastructure Investment ss.com/article/indi blueprint for tackling climate Change Select the correct answer using the
Bank a/india-news- change, emphasised eight key (UNFCCC). code given below.
(d) plan of action outlined by india/here-are- goals — sustainable lifestyles, (a) 1 and 2 only
the countries of the world indias-indc- cleaner economic (b) 2, 3 and 4 only
regarding Sustainable objectives-and- development, reducing (c) 3 and 4 only
Development Goals how-much-it-will- emission intensity of GDP, (d) 1, 2, 3 and 4
cost/ increasing the share of non-
fossil fuel based electricity,
enhancing carbon sink,
adaptation and mobilising
finance, technology transfer
and capacity building.
32 Economy Which one of the following is D E FCA Economic survey EN The Union Cabinet had given Recently Asked in Open Test conducted by
a purpose of ‘UDAY’, a its approval to a new scheme Union Vision IAS - Test 1789
scheme of the Government? http://www.thehin moved by the Ministry of cabinet had Which of the following statements
(a) Providing technical and du.com/business/I Power - Ujwal DISCOM given its is/are correct about recently
financial assistance to start- ndustry/governme Assurance Yojna or UDAY. approval. launched Ujwal DISCOM Assurance
up entrepreneurs in the field nts-uday-scheme- UDAY provides for the It is one of Yojna (UDAY) by Union
of renewable sources of may-light-up- financial turnaround and the government?
energy power- revival of Power Distribution important 1. It has been launched by Union
(b) Providing electricity to distribution- companies (DISCOMs), and scheme of Ministry of Power for financial
every household in the companies- importantly also ensures a Union restructuring of debt of power
countries by 2018 crisil/article809881 sustainable permanent government distribution companies.
(c) Replacing the coal-based 8.ece solution to the problem. to 2. It seeks to make DISCOMs
power plants with natural Under UDAY, state accelerates financially healthy with an
gas, nuclear, solar, wind and http://pib.nic.in/ne governments will take over 75 the process ambition of meeting target of

25 www.visionias.in ©Vision IAS


tidal power plants over a wsite/PrintRelease. per cent of the debt held by of reform 100% Village electrification.
period of time aspx?relid=130261 their discoms as of September across the Select the correct answer using the
(d) Providing for financial 30, 2015. Half the debt will be entire code given below.
turnaround and revival of taken over in 2015-16 and 25 power (a) 1 only
power distribution per cent in 2016-17. The sector. (b) 2 only
companies balance 25 per cent of the (c) Both 1 and 2
debt is to be serviced through (d) Neither 1 nor 2
state government-guaranteed
bonds issued by the discoms.
33 Current With reference to ‘IFC C M CAA http://pib.nic.in/ne EN Bonds are instruments of debt In 2015, IFC Asked twice in all India test series
Affairs Masala Bonds’, sometimes wsite/PrintRelease. - typically used by corporates issued the Tests - 1720,1768, 2059, 2067
seen in the news, which of aspx?relid=139028 to raise money from first Masala Masala Bonds recently been in
the statements given below investors. Masala bonds have bond listed news, refers to:
is/are correct? http://www.thehin to be explained in the context on the (a) Group of international spice
1. The International Finance du.com/business/I of Indian corporates raising London industries whose members can
Corporation, which offers ndustry/masala- money from overseas Stock issue bonds to each other.
these bonds, is an arm of the bonds-the-flavour- investors. Exchange. (b) Offshore rupee denominated
World Bank. of-the- The International Finance A number of bonds issued by Indian corporates
2. They are the rupee- moment/article79 Corporation (IFC), the Indian firms to overseas buyers.
denominated bonds and are 30287.ece investment arm of the World are gearing (c) Illegal bonds purchased by
a source of debt financing for Bank, issued a Rs. 1,000 crore up to raise Foreign Institutional Investors
the public and private sector, bond in November last year. funds from (FIIs).
Select the correct answer The purpose of the issue was issuing what (d) Inflation indexed bonds.
using the code given below. to fund infrastructure are known Test 1785
(a) 1 only projects in India. as ‘masala Which of the following correctly
(b) 2 only Companies including NTPC, bonds.’ defines 'Masala bonds'?
(c) Both 1 and 2 Neyveli Lignite Corporation, Coinciding (a) They are rupee-denominated
(d) Neither 1 nor 2 Power Finance Corporation, with Prime instruments sold only to offshore
Power Trading Corporation Minister investors by Indian corporates to
and Rural Electrification Narendra raise money from overseas.
Corporation are likely to Modi’s (b) They are dollar-denominated
launch these Masala Bonds recent visit instruments sold only to offshore
totalling $1 billion in the next to the UK investors by Indian corporates to
three or four months in the raise money from overseas.
UK to gauge the investor (c) They are rupee-denominated
appetite. The tenor of these instruments sold only to Indian
bonds is likely to be limited investors by offshore corporates to
to a band of five to seven raise money from India.
years and these are going to (d) They are dollar-denominated
be in smaller denominations instruments sold only to Indian
ranging from $150-250 investors by offshore corporates to

26 www.visionias.in ©Vision IAS


million. These will be subject raise money from India.
to decisions made by the
Boards of the PSU energy
companies.
34 Medieval Regarding the taxation C D F http://shodhganga RR Vijayanagara rulers followed Depth
History system of Krishna Deva, the .inflibnet.ac.in/bits an oppressive taxation policy. Knowledge
ruler of Vijayanagar, consider tream/10603/2560 The prosperity of the empire about
the following statements: 0/7/07_chapter%2 can be explained through history
1. The tax rate on land was 02.pdf their taxation principles in the
fixed depending on the following lines. They collected
quality of the land. variety of taxes ranging on
2. Private owners of the nature of lands.
workshops paid an industries The villages were classified
tax. into eight types for tax
Which of the statements purpose.
given above is/are correct? They were:
(a) 1 only 1)The brahmadeya villages
(b) 2 only 2) The devadana villages
(c) Both 1 and 2 3) The mathapura villages
(d) Neither 1 nor 2 4) The sarvamanya villages
5) The kodagi lands
6) Rakta-kodagal lands
7) Guttagi lands
8) The rest of the villages or
lands.
The first four types prevailed
prior to the Vijayanagar times.
Private owners also paid
industries taxes. Merchant
guilds were empowered to
levy taxes on goods
manufactured, sold or
transported
35 Culture Which one of the following B D F http://ccrtindia.go EM Kalidasa is the most Knowledge
books of ancient India has v.in/literaryarts.ph distinguished dramatist and about
the love story of the son of p his treatment of the rasa of literary
the founder of Sunga love in all its possible history
dynasty? manifestations in the three
(a) Swapnavasavadatta plays Malavikagnimitra
(b) Malavikagnimitra (Malavika and Agnimitra),
(c) Meghadoota Vikramorvasiya (Vikram and

27 www.visionias.in ©Vision IAS


(d) Ratnavali Urvasi) and Abhigyana
Shakuntala (the recognition of
Shakuntala) is unparalleled.
He is the poet of love and
beauty, and believes in the
affirmation of life, the joy of
which lies in pure, sacred and
ever-widening love
36 Economy In the context of which of A E CAA Economic Survey EM In WTO terminology, The WTO Cover in PT 365
the following do you subsidies in general are members All India Test series
sometimes find the terms http://indianexpre identified by “Boxes” which are 1723, 1774, 1891, 2063, 2064
‘amber box, blue box and ss.com/article/indi are given the colours of traffic thrashing Also covered in Open Test 1763
green box’ in the news? a/india- lights: green (permitted), out a work With reference to WTO
(a) WTO affairs others/govt-works- amber (slow down — i.e. be programme terminology subsidies are
(b) SAARC affairs to-finetune-farm- reduced), red (forbidden). In for the 10th identified by "Boxes" of different
(c) UNFCCC affairs subsidy-as-per- agriculture, things are, as Ministerial colors. Match the Colour Boxes in
(d) India-EU negotiations on wto/ usual, more complicated. The to be held in List I with their relevance in List II.
FTA Agriculture Agreement has no Nairobi in List I List II
http://www.thehin Red Box, although domestic December 1. Blue Box : A. Minimum
dubusinessline.co support exceeding the 2015 disruption of trade balance
m/economy/eu- reduction commitment levels 2. Amber Box : B. Subsidies that
canada-redflag- in the Amber Box is limit production
indias-crop-cover- prohibited; and there is a Blue 3. Green Box : C. Distort trade
scheme-at- Box for subsidies that are tied balance because they encourage
wto/article838187 to programmes that limit excessive production.
9.ece production. Select the correct answer using the
The 'amber box': For code given below.
agriculture, all domestic (a) 1-B, 2-A, 3-C
support measures considered (b) 1-A, 2-C, 3-B
to distort production and (c) 1-B, 2-C, 3-A
trade (with some exceptions) (d) None
fall into the amber box.
The 'green box': In order to
qualify for the "green box", a
subsidy must not distort
trade, or at most cause
minimal distortion. These
subsidies have to be
government-funded (not by
charging consumers higher
prices) and must not involve

28 www.visionias.in ©Vision IAS


price support.
The 'blue box': The blue box
is an exemption from the
general rule that all subsidies
linked to production must be
reduced or kept within
defined minimal ("de
minimis") levels. It covers
payments directly linked to
acreage or animal numbers,
but under schemes which
also limit production by
imposing production quotas
or requiring farmers to set
aside part of their land.
Countries using these
subsidies - and there are only
a handful - say they distort
trade less than alternative
amber box subsidies.
Currently, the only members
notifying the WTO that they
are using or have used the
blue box are: the EU, Iceland,
Norway, Japan, the Slovak
Republic, Slovenia, and the
US (now no longer using the
box).
37 Economy Which of the following is/are D M FA NCERT Book For EM The Capital Budget is an Basic Vision Ias All India tesr series:
included in the capital Class XII : account of the assets as well economic 1787
budget of the Government of Economics - as liabilities of the central and Which of the following are
India? Macroeconomics government, which takes into government recognized as Capital expenditures
1. Expenditure on acquisition Chapter 5 Page 63 consideration changes in budgeting in the Union Budget?
of assets like roads, capital. It consists of capital concept 1. Loans to state governments
buildings, machinery, etc. receipts and capital 2. Expenditure on the acquisition
2. Loans received from expenditure of the of land or a building
foreign governments government. This shows the 3. Investment in shares
3. Loans and advances capital requirements of the 4. Interest payments on debts
granted to the States and government and the pattern Select the correct answer using the
Union Territories of their financing The main code given below.
Select the correct answer items of capital receipts are (a) 1, 2 and 3 only

29 www.visionias.in ©Vision IAS


using the code given below. loans raised by the (b) 1, 2 and 4 only
(a) 1 only government from the public (c) 2 and 3 only
(b) 2 and 3 only which are called market (d) 1, 3 and 4 only
(c) 1 and 3 only borrowings, borrowing by the
(d) 1, 2 and 3 government from the Reserve
Bank and commercial banks
and other financial
institutions through the sale
of treasury bills, loans
received from foreign
governments and
international organisations,
and recoveries of loans
granted by the central
government. Capital
Expenditure: There are
expenditures of the
government which result in
creation of physical or
financial assets or reduction in
financial liabilities. This
includes expenditure on the
acquisition of land, building,
machinery, equipment,
investment in shares, and
loans and advances by the
central government to state
and union territory
governments, PSUs and other
parties
38 Environment What is/are the importance/ C D FA http://pib.nic.in/ne EN Desertification, along with Recently Asked twice in All India Test
importances of the ‘United wsite/erelcontent. climate change and the loss of ISRO-led series.
Convention to Combat aspx?relid=39572 biodiversity, were identified study Test – 1785
Desertification’? as the greatest challenges to analysed Which of the following statement
1. It aims to promote http://www.unccd. sustainable development satellite is not correct about United Nations
effective action through int/en/about-the- during the 1992 Rio Earth imageries of Convention to Combat
innovative national convention/Pages/ Summit. Established in 1994, the country Desertification?
programmes and supportive About-the- UNCCD is the sole legally over an (a) It is a not legally binding to its
international partnerships Convention.aspx binding international eight-year signatories.
2. It has a special/particular agreement linking period (b) It encourages the participation
focus on South Asia and http://www.unesc environment and shows that of local people in combating

30 www.visionias.in ©Vision IAS


North Africa regions, and its o.org/mab/doc/ek development to sustainable nearly 30 desertification.
Secretariat facilitates the ocd/chapter14.ht land management. per cent per (c) It was established in
allocation of major portion of ml The Convention addresses cent of the accordance to the
financial resources to these specifically the arid, semi-arid country’s recommendations of 1992 Rio
regions. and dry sub-humid areas, total Earth Summit.
3. It is committed to bottom- known as the drylands, geographical (d) India is a signatory to the
up approach, encouraging where some of the most area is convention. Test -
the participation of local vulnerable ecosystems and undergoing 1725,1776,1895,2070 With
people in combating the peoples can be found. degradation. reference to the United Nations
desertification. The Convention's 195 parties Convention to Combat
Select the correct answer work together to improve the Desertification (UNCCD), consider
using the code given below. living conditions for people in the following statements:
(a) 1 only drylands, to maintain and 1. It addresses the issue of drought
(b) 2 and 3 only restore land and soil in arid and semi-arid areas.
(c) 1 and 3 only productivity, and to mitigate 2. It is a legally binding
(d) 1, 2 and 3 the effects of drought. international agreement.
The UNCCD is particularly 3. India is not a signatory to the
committed to a bottom-up convention.
approach, encouraging the Which of the statements given
participation of local people above is/are correct?
in combating desertification (a) 1 and 2 only
and land degradation. (b) 2 and 3 only
The UNCCD secretariat (c) 1 and 3 only
facilitates cooperation (d) 1, 2 and 3
between developed and
developing countries,
particularly around
knowledge and technology
transfer for sustainable land
management.
39 Current Recently, which one of the D E CA http://www.thehin EN The Executive Board of the Recently Tests - 1721,1769, 2067 , 2093
Affairs following currencies has du.com/business/E International Monetary Fund chineese (PT365)
been proposed to be added conomy/imf- decided to include the currency Which of the following currencies
to the basket of IMF’s SDR? names-chinese- Chinese currency, the Renminbi form the part of special drawing
(a) Rouble renminbi-yuan- renminbi (yuan), into its included right (SDR) of IMF?
(b) Rand global-reserve- basket of currencies that into 1. Chinese Renminbi
(c) Indian Rupee currency/article79 make up the IMF’s Special 2. Japanese Yen
(d) Renminbi 33846.ece Drawing Right (SDR). The 3. British Pound
decision was taken during the 4. Russian Rouble
IMF’s five-yearly review of the Select the correct answer using the
basket of currencies. code given below.

31 www.visionias.in ©Vision IAS


SDRs are artificial currency (a) 1 and 2 only
created to augment (b) 1, 2 and 3 only
international liquidity. It is (c) 3 and 4 only
neither a currency nor a claim (d) 1, 2, 3 and 4
on IMF rather it supplements Which of the following is not
the existing reserves of correct regarding SDRs?
member countries of IMF. To (a) IMF decided to include Chinese
be included as SDR a currency renminbi (RMB) in the basket of
must be freely usable, widely currencies that make up Special
used and widely traded. U.S. Drawing Right (SDR) with effect
dollar, the euro, British pound from October 1, 2016.
and the Japanese yen form (b) It is an international currency
the SDR basket. IMF has created by the IMF in 1969.
decided to include Yuan in the (c) SDRs can be exchanged for
SDR basket with effect from freely usable currencies.
October 1, 2016. The (d) It was created in response to
respective weights of the U.S. concerns about the limitations of
dollar, euro, Chinese gold and dollars as the sole means
renminbi, Japanese yen, and of settling international accounts.
pound sterling are 41.73
percent, 30.93 percent, 10.92
percent, 8.33 percent.
40 Economy With reference to the C M FCA http://www.thehin RR The IMFC advises and reports
International Monetary and dubusinessline.co to the IMF Board of
Financial Committee (IMFC), m/economy/indian Governors on the supervision
consider the following -economy-clearly- and management of the
statements: on-recovery-path- international monetary and
1. IMFC discusses matters of jaitley/article7118 financial system, including on
concern affecting the global 986.ece responses to unfolding events
economy, and advises the http://www.imf.or that may disrupt the system
International Monetary Fund g/external/np/exr/ although the IMFC has no
(IMF) on the direction of its facts/groups.htm formal decision-making
work. powers, in practice, it has
2. The World Bank become a key instrument for
participates as observer in providing strategic direction
IMFC’s meetings. to the work and policies of the
Which of the statements IMF. A number of
given above is/are correct? international institutions,
(a) 1 only (b) 2 only including the World Bank,
(c) Both 1 and 2 participate as observers in the
(d) Neither 1 nor 2 IMFC’s meetings.

32 www.visionias.in ©Vision IAS


33 www.visionias.in ©Vision IAS
41 Polity and ‘Rashtriya Garima Abhiyaan’ C M CAA http://pib.nic.in/ne EN Campaign has started in 2001 Socio-
Governance is a national campaign to wsite/mbErel.aspx known as ‘Rashtriya Garima Economic
(a) rehabilitate the homeless ?relid=126057 Abhiyan’, (National Campaign Caste
and destitute persons and for Dignity and Elimination of Census data
provide them with suitable http://www.thehin Manual Scavenging) has released on
sources of livelihood dubusinessline.co proven to be a very innovative July 3
(b) release the sex workers m/news/variety/fig and effective program to reveals that
from their practice and hting-for- eliminate manual scavenging 1, 80, 657
provide them with dignity/article7342 practice. Campaign has households
alternative sources of 754.ece liberated 11,000 women are engaged
livelihood manual scavengers in various in practice
(c) eradicate the practice of parts of India and urged them of manual
manual scavenging and to give up this caste-based, scavenging.
rehabilitate the manual unconstitutional practice. In
scavengers its efforts campaign attain to
(d) release the bonded social, economic, political and
labourers from their cultural rehabilitation of
bondage and rehabilitate Manual Scavenger.
them
42 Medieval With reference to the C D F NCERT Themes in EM The twelfth century witnessed to check in -
History cultural history of medieval Indian history part the emergence of a new depth
India, consider the following -2(page no.-147) movement in Karnataka, led knowledge
statements: by a Brahmana named of history
1. Siddhas (Sittars) of Tamil NCERT our past 2- Basavanna (1106-68) who was
region were monotheistic Bhakti movements initially a Jaina and a minister
and condemned idolatry. in the court of a Chalukya
2. Lingayats of Kannada king. His followers were
region questioned the theory known as Virashaivas (heroes
of rebirth and rejected the of Shiva) or Lingayats
caste hierarchy. (wearers of the linga).The
Which of the statements Lingayats challenged the idea
given above is/are correct? of caste and the “pollution”
(a) 1 only attributed to certain groups
(b) 2 only by Brahmanas. They also
(c) Both 1 and 2 questioned the theory of
(d) Neither 1 nor 2 rebirth. These won them
followers amongst those who
were marginalised within the
Brahmanical social order.
Siddhas (Sittars) Saiva school
in Tamil Nadu which held the

34 www.visionias.in ©Vision IAS


monotheistic puritan creed
and roundly condemned
idolatry, there history are not
known they seem to be
known as most active during
16th and 17th century.
43 Economy Which of the following best D M FA http://www.thehin EN Import cover is an important Recently
describes the term ‘import du.com/business/E indicator of the stability of the government
cover’, sometimes seen in conomy/import- currency. It measures the official data
the news? cover-increases-to- number of months of money shows that
(a) It is the ratio of value of 98-months-says- available in the national bank India
imports to the Gross rbi/article8093212. to cover the cost of imports or imports
Domestic Product of a ece the stock of foreign exchange cover
country reserves in terms of months increae to
(b) It is the total value of of retained imports of goods 10.9 months
imports of a country in a as at end of year. During the which
year currency crisis of 2013, when signifies that
(c) It is the ratio between the foreign exchange reserves fell Indian
value of exports and that of to around $275 billion, import foreign
imports between two cover dipped to around seven currency
countries months. reserves had
(d) It is the number of increased in
months of imports that could recent past.
be paid for by a country’s
international reserves
44 Current Consider the following pairs: C M CAA http://www.thehin EN The Madhesi also referred to All the three .
Affairs Community sometimes du.com/news/inter as Teraibasi Nepali are an community
mentioned in the news In the national/india- indigenous ethnic group of are in news
affairs of courts-radical- Nepalese people who are for recent
1. Kurd : Bangladesh madhesis-from- natives of the Madhesh plains past,
2. Madhesi : Nepal nepal/article81639 of Southern Nepal in Terai Like kurds in
3. Rohingya : Myanmar 11.ece belt of South Asia. Iraq and
Which of the pairs given Between 25 and 35 million Syria are in
above is/are correctly http://www.thehin Kurds inhabit a mountainous news for
matched? du.com/opinion/o region straddling the borders their
(a) 1 and 2 p-ed/turkeys-war- of Turkey, Iraq, Syria, Iran and massacre by
(b) 2 only on-the- Armenia. They make up the ISIS
(c) 2 and 3 kurds/article80653 fourth-largest ethnic group in terrorists.
(d) 3 only 03.ece the Middle East, but they Madhesi
have never obtained a peoples are
permanent nation state. in news

35 www.visionias.in ©Vision IAS


http://www.thehin The Rohingya are an ethnic because of
du.com/news/nati Muslim minority in the consitutiona
onal/report-puts- majority Buddhist Myanmar. l deadlock
spotlight-on- Many of their enemies refuse and conflict
status-of- to acknowledge that the between
rohingyas/article82 Rohingya are an ethnically them and
19725.ece distinct group. They claim Nepali
instead that the Rohingya are government
Bengali and that their Rohingya
presence in Myanmar is the muslims in
result of illegal immigration myanmar
(more on that later). The are in news
Rohingya, for their part, claim for their
to be pre-colonial residents of conflict with
Myanmar’s Rakhine state, the Myanmar
Middle East Institute explains, majority
with the earliest known buddhist
appearance of the term population
Rohingya in 1799 and
thousands
of ethnic
Rohingya
are fleeing
persecution.
45 Current With reference to B M FCA http://www.thehin EN OPCW is the Hague-based It was in Tests - 1715 and 1767
Affairs ‘Organiation for the du.com/news/inter autonomous body, which news due to With reference to the Organization
Prohibition of Chemical national/world/op works within the framework reports of for the Prohibition of Chemical
Weapons (OPCW)’, consider cw-is-upholding- of the United Nations, was Chemical Weapons consider the following
the following statements: nobels- established in 1997 by the weapons statements:
1. It is an organization of vision/article52249 Chemical Weapons used by ISIS 1. OPCW is an intergovernmental
European Union in working 90.ece Convention to carry out its as well as organisation, headquatered in
relation with NATO and mandate. Assad Hague, Netherlands.
WHO. OBJECTIVES: regime in 2. It is the implementing body of
2. It monitors chemical OPCW Member States share Syria. It was the Chemical Weapons Convention
industry to prevent new the collective goal of awarded the (CWC)
weapons from emerging. preventing chemistry from Nobel Peace 3. It was awarded Nobel Peace
3. It provides assistance and ever again being used for Prize in Prize in 2013.
protection to States (Parties) warfare, thereby 2013 for its Which of the statements given
against chemical weapons strengthening international extensive above is/are correct?
threats. security. To this end, the efforts to (a) 1 and 3 only
Which of the statements Convention contains four key eliminate (b) 2 and 3 only

36 www.visionias.in ©Vision IAS


given above is/are correct? provisions: chemical (c) 3 only
(a) 1 only destroying all existing weapons. (d) 1, 2 and 3
(b) 2 and 3 only chemical weapons under
(c) 1 and 3 only international verification by
(d) 1, 2 and 3 the OPCW
monitoring chemical industry
to prevent new weapons from
re-emerging;
providing assistance and
protection to States Parties
against chemical threats; and
fostering international
cooperation to strengthen
implementation of the
Convention and promote the
peaceful use of chemistry.
46 Current With referene to ‘Pradhan B M CA http://pib.nic.in/ne EN Pradhan Mantri Fasal Bima Recently it Asked in All India Test series Tests
Affairs Mantri Fasal Bima Yojana’, wsite/PrintRelease. Yojana (PMFBY) is the new was - 1723,1771,1894,2069
consider the following aspx?relid=134432 crop damage insurance launched by With reference to Pradhan Mantri
statements: scheme that has been central Fasal Bima Yojana, consider the
1. Under this scheme, http://www.thehin approved by the Union government following statements:
farmers will have to pay a du.com/news/nati Cabinet in January 2016. It will to enhance 1. It covers localized calamities
uniform premium of two onal/govt- replace the existing two crop insurance including hailstorms, unseasonal
percent for any crop they approves-new- insurance schemes National coverage to rains, landslides and inundation.
cultivate in any reason of the crop-insurance- Agricultural Insurance Scheme more crop 2. It does not cover nuclear risks
year. plan-for- (NAIS) and Modified NAIS. The area to and fields destroyed by domestic
2. This scheme covers post- farmers/article810 new scheme will come into protect or wild animals.
harvest losses arising out of 2137.ece force from the Kharif season farmers 3. It proposes mandatory use of
cyclones and unseasonal starting in June this year. from remote sensing, smart phones and
rains. One of the highlights included vagaries of drones for quick estimation of crop
Which of the statements there will be a uniform monsoon loss.
given above is/are correct? premium of only 2% to be and this 4. Post-harvest losses coverage is
(a) 1 only paid by farmers for all Kharif schemes is included under this scheme.
(b) 2 only crops and 1.5% for all Rabi radically Which of the statements given
(c) Both 1 and 2 crops. In case of annual departure above is/are correct?
(d) Neither 1 nor 2 commercial and horticultural from earlier (a) 1 and 2 only
crops, the premium to be paid crop in (b) 1, 3 and 4 only
by farmers will be only 5%. surance (c) 2, 3 and 4 only
Also POST-HARVEST LOSSES schemes. (d) 1, 2, 3 and 4
includes (individual farm
basis): Coverage is available

37 www.visionias.in ©Vision IAS


upto a maximum period of 14
days from harvesting for
those crops which are kept in
"cut & spread" condition to
dry in the field after
harvesting, against specific
perils of cyclone / cyclonic
rains, unseasonal rains
throughout the country.
Hence only statement 2 is
correct.
47 Environment In which of the following D M CA http://www.thehin EN The great hornbill, a
regions of India are you most du.com/news/citie magnificent bird reaching a
likely to come across the s/Coimbatore/gro length of three-and-a-half
‘Great Indian Hornbill’ in its wing-threat-to- feet, is distinguished by a big
natural habitat? great-indian- yellow beak with a casque and
(a) Sand deserts of northwet hornbills/article71 striking tail feathers the
India 37163.ece magnificent birds which were
(b) Higher Himalayas of a common sight in the
Jammu and Kashmir Western Ghats are rarely
(c) Salt marshes of western seen, due to deforestation.
Gujarat habitat loss threatens its
(d) Western Ghats future, and hunting has
depleted populations. It is
protected at the highest level
under Schedule I of the
Wildlife Protection Act, but
that can do little to save the
habitat.
48 Current Which of the following are A M FCA http://nmcg.nic.in/ EN The Central Government set National
Affairs the key features of ‘National ngrbaread.aspx up the ‘National Ganga River Mission for
Ganga River Basin Authority Basin Authority’ (NGRBA) in Clean Ganga
(NGRBA)? http://pib.nic.in/ne 2009 and mandated it to take (NMCG) is
1. River basin is the unit of wsite/PrintRelease. up regulatory and the
planning and management. aspx?relid=137894 developmental functions with implementa
2. It spearheads the river sustainability needs for tion wing of
conservation efforts at the effective abatement of National
national level. pollution and conservation of Ganga River
3. One of the Chief Ministers the river Ganga by adopting a Basin
of the States through which river basin approach for Authority
the Ganga flows becoms the comprehensive planning and (NGRBA).

38 www.visionias.in ©Vision IAS


Chairman of NGRBA on management.
rotation basis. The Ministry of Water
Select the correct answer Resources, River Development
using the code given below. and Ganga Rejuvenation
(a) 1 and 2 only (MoWR, RD & GR) is the nodal
(b) 2 and 3 only Ministry for the NGRBA. The
(c) 1 and 3 only authority is chaired by the
(d) 1, 2 and 3 Prime Minister and has as its
members the Union Ministers
concerned, the Chief
Ministers of the States
through which Ganga flows,
viz., Uttarakhand, Uttar
Pradesh, Bihar, Jharkhand and
West Bengal, among others.
This initiative is expected to
rejuvenate the collective
efforts of the Centre and the
States for cleaning the river.
NGRBA functions include
development of a Ganga River
Basin Management Plan,
regulation of activities aimed
at prevention, control and
abatement of pollution, to
maintain water quality and to
take measures relevant to the
river ecology in the Ganga
basin states. It is mandated to
ensure the maintenance of
minimum ecological flows in
the river Ganga and abate
pollution through planning,
financing and execution of
programmes including that of
1) Augmentation of Sewerage
Infrastructure
2) Catchment Area Treatment
3) Protection of Flood Plains
4) Creating Public Awareness

39 www.visionias.in ©Vision IAS


49 Current Why does the Government B M FA Economic survey EM Urea is an important supplier Government Tests - 1722, 1770, 2063
Affairs of India promote the use of (2015-16) of nitrogen, which is promoted Which of the following best
‘Neem-coated Urea’ in necessary for the neem coted explains the advantage of neem-
agriculture? development of plants. But urea to coated urea over normal urea?
(a) Release of Neem oil in the only 30-40 per cent of prevent the (a) Neem checks the loss of
soil increases nitrogen nitrogen present in the urea is problem of nitrogen by slowing nitrate
fixation by the soil utilised by crops. The rest gets uncontrolled formation.
microorganisms degraded. use of (b) Neem coated urea enhances
(b) Neem coating slows When ordinary urea is chemical the rate of absorption of nitrogen.
down the rate of dissolution applied, it gets converted to fertilizer and (c) Neem coated urea increases
of urea in the soil ammonium carbamate. Some it was soil’s capacity to retain nitrogen.
(c) Nitrous oxide, which is a of this gets converted to emphasise (d) Presence of neem increases the
greenhouse gas, is not at all ammonia gas in what is called in Economic life span of plants.
released into atmosphere by ammonia volatilisation. survey.
crop fields About 8-10 per cent nitrogen
(d) It is a combination of a is lost during volatilization.
weedicide and a fertilizer for The rest of the ammonium
particular crops carbamate undergoes
chemical transformation and
nitrates are formed. Some of
these are absorbed by the
plants. The rest are either
leached into the underground
water or are denitrified to
gaseous nitrogen and nitrous
oxide under anaerobic
conditions (absence of
oxygen). Neem has properties
that check nitrogen loss at
each stage. It slows down the
process of nitrate formation
and hence excess nitrate is
not available for
denitrification.
When farmers use this urea,
the slow release of nitrogen
helps the fertility of the soil.
Therefore, there will be more
yield to the extent of 15-20%.
Also, the higher usage of
neem-coated urea would

40 www.visionias.in ©Vision IAS


check diversion of urea for
industrial use. Neem coated
urea also works as insecticide.
50 Polity and Consider the following D D FA Indian Public RR The incumbent to the post of To check the
Governance statements: Administration(thir Chief Secretary is chosen by deep
1. The Chief Secretary in a d edition) and appointed by the Chief understandi
State is appointed by the Ramesh k Arora Minister and the Ministers of ngs of the
Governor of that State. and Rajni Goyal the State. The trend is that the concept of
2. The Chief Secretary in a Chapter 18(The Chief Minister consults the Polity.
State has a fixed tenure. Chief Secretary and Union Government regarding
Which of the statements State Secretariat) the appointment of the Chief
Secretary but, this consultation
given above is/are correct? Page no.-335-336
is not obligatory.
(a) 1 only Supreme Court of
There is no fixed tenure for the
(b) 2 only India
post of Chief Secretary.
(c) Both 1 and 2
Administrative reform
(d) Neither 1 nor 2 E. P. Royappa vs commission in 1969 had
State Of Tamil recommended that a Chief
Nadu & Anr on 23 secretary should have a
November, 1973 minimum tenure of three to
four years.
51 Current With reference to ‘Stand Up C M CA http://www.thehin EN "Stand Up India Scheme" was Constantly Tests- 1724,1775,1891,2063
Affairs India Scheme’, which of the du.com/news/nati initiated to promote in news "Stand up India Scheme" was
following statements is/are onal/modi-to- entrepreneurship among launched under the broader theme
correct? launch-stand-up- Scheduled Castes, Schedule of "Start Up India, Stand Up India"
1. Its purpose is to promote india-scheme- Tribes and Women. to promote entrepreneurship
entrepreneurship among tomorrow/article8 The scheme will provide among:
SC/ST and women 430358.ece financial aid through the Small 1. Scheduled Castes
entrepreneurs. http://pib.nic.in/ne Industries Development Bank 2. Scheduled Tribes
2. It provides for refinance wsite/PrintRelease. of India (SIDBI) with an initial 3. Differently-abled
through SIDBI. aspx?relid=138523 amount of Rs 10,000 crore. 4. Women
Select the correct answer The system will personally 5. Minorities
using the code given below. guide each entrepreneur Select the correct answer using the
(a) 1 only through the pre-loan and code given below.
(b) 2 only operational phases. (a) 1, 2 and 4 only
(c) Both 1 and 2 The scheme will also (b) 1, 3 and 5 only
(d) Neither 1 nor 2 familiarise the entrepreneurs (c) 2, 4 and 5 only
with factoring services, e- (d) 1, 2, 3, 4 and 5
market places and registration
with online platforms and
other aspects of web
entrepreneurship.
41 www.visionias.in ©Vision IAS
42 www.visionias.in ©Vision IAS
52 Environment The FAO accords the status B M FCA http://www.thehin EN Statement 1 is incorrect. Test- 2139( Open Test 5)
of ‘Globally Important du.com/sci- GIAHS aims to protect This region is a delta region of
Agricultural Heritage System tech/ecofriendly- traditional agricultural about 900 sq. km situated in the
(GIAHS)’ to traditional technologies-fetch- systems. It will not provide west coast of Kerala State, India.
agricultural systems. What is better-results-in- any modern technology and The farming System practiced here
the overall goal of this rice- training. is unique as it is the only system in
initiative? production/article Statement 2 is correct. GIAHS India that practices rice cultivation
1. To provide modern 7154833.ece aims to identify and safeguard below sea level. It is one of the
technology, training in eco-friendly traditional farm Globally Important Agricultural
modern farming methods http://www.fao.or practices and their associated Heritage Systems (GIAHS) sites in
and financial support to local g/giahs/giahs/giahs landscapes, agricultural India." The above passage
communities of identified -about/en/ biodiversity and knowledge describes which of the following
GIAHS so as to greatly systems of the local places?
enhance their agricultural communities. (a) Koratpur
productivity. Statement 3 is also incorrect. (b) Pampore
2. To identify and safeguard There is no such provision. (c) Kuttanad
eco-friendly traditional farm (d) Munnar
practices and their
associated landscapes,
agricultural biodiversity and
knowledge systems of the
local communities
3. The provide Geographical
Indication status to all the
varieties of agricultural
produce in such identifies
GIAHS
Select the correct answer
using the code given below.
(a) 1 and 3 only (b) 2 only
(c) 2 and 3 only (d) 1, 2 and 3
53 Geography Which of the following is/are D M F NCERT 11, India EM All the given rivers Dibang, Concept of
tributary/ tributaries of physical Kameng and Lohit are the Geography
Brahmaputra? environment, page tributaries of Brahmaputra.
1. Dibang 26
2. Kameng
3. Lohit
Select the correct answer
using the code given below.
(a) 1 only (b) 2 and 3 only
(c) 1 and 3 only (d) 1, 2 and 3

43 www.visionias.in ©Vision IAS


54 Economy The term ‘Core Banking A M FCA http://www.thehin EN Only statement 1 is correct. India post's
Solution’ is sometimes seen du.com/news/nati Core banking solution enables IT
in the news. Which of the onal/tamil- customers to operate their modernisati
following statements best nadu/post-offices- accounts, and avail banking on project
describes/describe this in-south-migrating- services from any branch of
term? to-core-banking- the bank on CBS network,
1. It is a networking of a solutions/article82 regardless of where they oepn
bank’s branches which 21039.ece and maintain their account.
enables customers to
operate their accounts from
any branch of the bank on its
network regardless of where
they open their accounts.
2. It is an effort to increase
RBI’s control over
commercial banks through
computerization.
3. It is a detailed procedure
by which a bank with huge
non-performing assets is
taken over by another bank.
Select the correct answer
using the code given below.
(a) 1 only
(b) 2 and 3 only
(c) 1 and 3 only
(d) 1, 2 and 3
55 Environment Consider the following pairs: C E F Economic survey EM The Clean Development Climate Test – 1784
Terms sometimes seen in (2015-16) Vol 2 pg. Mechanism (CDM), created change Consider the following statements
the news Their origin 187 multilaterally under the negotiations regarding the Clean Development
1. Annex-I Countries : http://unfccc.int/k UNFCCC is one of the Mechanism (CDM):
Cartagena Protocol yoto_protocol/mec mitigation instruments under 1. It is one of mitigation centric
2. Certified Emissions hanisms/clean_dev the Kyoto Protocol. instruments created under the
Reductions : Nagoya Protocol elopment_mechani The Clean Development UNFCCC.
3. Clean Development sm/items/2718.ph Mechanism (CDM), defined in 2. Certified emission reduction
Mechanisms : Kyoto Protocol p Article 12 of the Protocol, (CER) credits earned under it can
Which of the pairs given allows a country with an be counted towards meeting Kyoto
above is/are correctly emission-reduction or targets.
matched? emission-limitation Which of the statements given
(a) 1 and 2 only commitment under the Kyoto above is/are correct?
(b) 2 and 3 only Protocol (Annex B Party) to (a) 1 only

44 www.visionias.in ©Vision IAS


(c) 3 only implement an emission- (b) 2 only
(d) 1, 2 and 3 reduction project in (c) Both 1 and 2
developing countries. Such (d) Neither 1 nor 2
projects can earn saleable Test – 1726, 1777, 2069
certified emission reduction With reference to Nagoya
(CER) credits, each equivalent Protocol, consider the following
to one tonne of CO2, which statements:
can be counted towards 1. It saves source countries from
meeting Kyoto targets. all forms of foreign bioprospecting.
Annex-I countries are parties 2. It is a legally binding treaty.
to Kyoto protocol. 3. It covers traditional knowledge
(TK) associated with genetic
resources.
Select the correct answer using the
code given below.
(a) 1 and 2 only
(b) 1 and 3 only
(c) 2 only
(d) 2 and 3 only

56 Science and In the context of the B D CA http://www.thehin EN Bioinformatics refers to the Recent
Technology developments in du.com/news/nati application of computer developmen
Bioinformatics, the term onal/karnataka/ge technology to the ts in
‘transcriptome’, sometimes nome-of-the-asian- management of biological technology
seen in the news, refers to elephant- information.
(a) a range of enzymes used sequenced-for-the- Transcriptome refers to the
in genome editing first-time-in- full range of messenger RNA
(b) the full range of mRNA india/article80060 (mRNA) molecules expressed
molecules expressed by an 26.ece from the genes of an
organism organism.
(c) the description of the
mechanism of gene
expression
(d) a mechanism of genetic
mutations taking place in
cells
57 Current ‘Mission Indradhanush’ A E CA http://www.thehin EN Mission Indradhanush: It was was Covered in PT365 material - Social
Affairs launched by the Government du.com/news/nati launched by Ministry of constantly in schemes
of India pertains to onal/kerala/missio Health and Family Welfare on news Tests - 1724,1775, 1891, 2065
(a) immunization of children n- 25th Dec 2014. The objective Which of the following diseases

45 www.visionias.in ©Vision IAS


and pregnant women indradhanush/artic is to ensure that all children are parts of 'Mission
(b) construction of smart le7732816.ece under the age of two years as Indradhanush'?
cities across the country well as pregnant women are 1. Diptheria
(c) India’s own search for the http://pib.nic.in/ne fully immunized with seven 2. Pertussis
Earth-like planets in outer wsite/PrintRelease. vaccine preventable diseases. 3. Polio
space aspx?relid=138342 By 2020, provide vaccination 4. Tuberculosis
(d) New Educational Policy to all children who are either 5. Chicken Pox
unvaccinated or partially 6. Dengue
vaccinated against: Select the correct answer using the
Diptheria, code given below.
Pertussis (Whooping cough) (a) 1, 2 and 3 only
Tetanus, (b) 1, 2, 3 and 4 only
Tuberculosis, (c) 4, 5 and 6 only
Polio, (d) 1, 2, 3, 4, 5 and 6
Measles and
Hepatitis B
The aim is to reach atleast
90% vaccination. At present
the immunisation is around
65%.

58 Environment Which of the following best C M FCA http://pib.nic.in/ne EN The National Mission for was Covered in PT365 material –
describes/ describe the aim wsite/PrintRelease. Green India (GIM) is one of constantly in Environment
of ‘Green India Mission’ of aspx?relid=128649 the eight Missions outlined news Tests - 1725, 1776
the Government of India? under the National Action With reference to the National
1. Incorporating http://www.envfor Plan on Climate Change Mission for Green India (GIM),
environmental benefits and .nic.in/major- (NAPCC). It aims at protecting; consider the following statements:
costs into the Union and initiatives/national restoring and enhancing 1. It is a mission under the
State Budgets thereby -mission-green- India’s diminishing forest National Action Plan on Climatic
implementing the ‘green india-gim cover and responding to Change (NAPCC).
accounting’ climate change by a 2. It will be implemented on both
2. Launching the second combination of adaptation public and private lands.
green revolution to enhance and mitigation measures. 3. It has been merged with
agricultural output so as to It envisages a holistic view of MGNREGA to improve the
ensure food security to one greening and focuses on country’s forest cover.
and all in the future multiple ecosystem services, Which of the statements given
3. Restoring and enhancing especially, biodiversity, water, above is/are correct?
forest cover and responding biomass, preserving (a) 1 only
to climate change by a mangroves, wetlands, critical (b) 1 and 2 only
combination of adaptation habitats etc. along with (c) 2 and 3 only
and mitigation measures carbon sequestration as a co- (d) 1, 2 and 3

46 www.visionias.in ©Vision IAS


Select the correct answer benefit. ANSWER – D
using the code given below. Statements 1 and 2 are not Which of the following statements
(a) 1 only the aims of Green India related to the Green India Mission
(b) 2 and 3 only Mission. (GIM) is/are correct?
(c) 3 only 1. It is a part of the National Action
(d) 1, 2 and 3 Plan on Climate Change.
2. The approval for alternate
energy devices using LPG are given
under GIM.
Select the correct answer using the
code given below.
(a) 1 only
(b) 2 only
(c) Both 1 and 2
(d) Neither 1 nor 2
ANSWER - C
59 Science and With reference to pre- C M CAA http://www.thehin EN Every pre-packaged items in was in news
Technology packaged items in India, it is du.com/opinion/o India the following in relation
mandatory to the p-ed/labelling-to- information on the label: to Maggi
manufacturer to put which take-the-pinch- 1. Name of the food issue and
of the following information out-of- 2. list of ingredients cancer
on the main label, as per the salt/article8801650 3. declaration of food causing
Food Safety and Standards .ece additives chemicals in
(Packaging and Labelling) 4. Net quantity or net weight bread
Regulations, 2011? http://www.fssai.g 5. Batch identification
1. List of ingredients ov.in/Portals/0/Pdf 6. Name and address of the
including additives /covering%20letter manufacturer
2. Nutrition information %20for%20draft%2 7. Date Marking
3. Recommendation, if any, 0regulation.pdf 8. Veg/ Non veg declaration
made by the medical
profession about the
possibility of any allergic
reactions
4. Vegetarian/non-
vegetarian
Select the correct answer
using the code given below.
(a) 1, 2 and 3
(b) 2, 3 and 4
(c) 1, 2 and 4
(d) 1 and 4 only

47 www.visionias.in ©Vision IAS


60 Current ‘Project Loon’, sometimes B E CA http://www.thehin EN Project Loon is a research and was Covered in PT365 material -
Affairs seen in the news, is related du.com/opinion/o development project being constantly in Science and Technology
to p-ed/its-a-bird-its- developed by Google X with news Tests - 1763, 1785
(a) waste management a-plane-its-the- the mission of providing With reference to 'Project Loon'
technology loon/article800865 Internet access to rural and consider the following statements:
(b) wireless communication 2.ece remote areas. The project 1. It proposes to provide internet
technology uses high-altitude balloons connectivity to remote areas using
(c) solar power production placed in the stratosphere at balloons floating in troposphere.
technology an altitude of about 18 km (11 2. It aims to tap unused TV
(d) water conservation mi) to create an aerial spectrum to provide internet
technology wireless network with up to access.
4G-LTE speeds. 3. The project is being
By partnering with implemented by Google Inc.
Telecommunications Which of the statements given
companies to share cellular above is/are correct?
spectrum Google is trying to (a) 1 and 3 only
enable people to connect to (b) 2 only
the balloon network directly (c) 2 and 3 only
from their phones and other (d) 3 only
LTE-enabled devices. ANSWER – D
Microsoft is planning to use Which of the following projects
unused TV spectrum to is/are related to providing last mile
provide internet access in internet connectivity?
remote areas. 1. Project Loon
2. Aquila
3. White Fi
Select the correct answer using the
code given below.
(a) 1 only
(b) 1 and 3 only
(c) 2 and 3 only
(d) 1, 2 and 3
ANSWER - D
61 Current ‘Net metering’ is sometimes A M CA http://www.thehin EN It is the mechanism of selling part of
Affairs seen in the news in the du.com/business/I the surplus solar power where India's
context of promoting the ndustry/metering- meters measure the solar Green
(a) production and use of policy-hindering- power exported to the grid; initiatives/e
solar energy by the rooftop-solar- the consumer is given credit nvironment
households/consumers sector-cleanmaxs- for the exported power, friendly
(b) use of piped natural gas jain- lightening the bill. strategies
in the kitchens of houeholds says/article854209

48 www.visionias.in ©Vision IAS


(c) installation of CNG kits in 6.ece
motor-cars
(d) installation of water
meters in urban households
62 Economy India’s ranking in the ‘Ease of C E CA http://www.thehin EN World bank's Ease of doing was Tests - 1779, 2069
Doing Business Index’ is du.com/news/nati business report, India's constantly in India has improved performance in
sometimes seen in the news. onal/india-moves- position has improved from news 'Ease of doing business' index
Which of the following has up-in-world-bank- 142 to 130. This is on account owing to improvement in which of
declared that ranking? doing-business- of ease of starting a business, the following indicators?
(a) Organization for 2016- dealing with construction 1. Starting a business
Economic Cooperation and ranking/article781 permit and getting electricity 2. Getting credit
Development (OECD) 1248.ece permits. Now, companies can 3. Dealing with construction
(b) World Economic Forum get connected to the grid and permit
(c) World Bank get on with their business, 14 4. Getting electricity
(d) World Trade Organization days sooner than before. The Select the correct answer using the
(WTO) number of days it takes to code given below.
start a new business has gone (a) 1, 3 and 4
up marginally from last year, (b) 1, 2 and 4
from 28.4 to 29 (c) 2 and 3
Credit is one area where India (d) 1, 2, 3 and 4
performs poorly and needs to
improve. Access to credit and
ease of paying taxes has
worsened, according to the
World Bank’s Doing Business
Report 2016.
63 Medieval Banjaras during the medieval D M F NCERT Class VII- Ch EM There were many kinds of to check
History period of Indian history were -6 and 7th: traders in medieval period basic
generally TOWNS, TRADERS these included the Banjaras /fundament
(a) agriculturists AND .They were the most al
(b) warriors CRAFTSPERSONS important trader nomads. knowledge
(c) weavers Their caravan was called
(d) traders tanda. Sultan Alauddin Khalji
used the Banjaras to transport
grain to the city markets.
Emperor Jahangir wrote in his
memoirs that the Banjaras
carried grain on their bullocks
from different areas and sold
it in towns.

49 www.visionias.in ©Vision IAS


64 Ancient Who of the following had B M F NCERT 12 - Themes EM It was James Prinsep in 1837 to check
History first deciphered the edicts of in Indian history, who succeeded in deciphering basic
Emperor Ashoka? page 28 an ancient inscription on a fundamenta
(a) Georg Buhler large stone pillar in Delhi l knowledge
(b) James Prinsep which was the edict of
(c) Max Muller Emperor Ashoka.
(d) William Jones
65 Polity and With reference to the ‘Gram B M F India year book, EM Gram Nyayalayas are mobile Gram Covered in tests 1714, 1765, 1789
Governance Nyayalaya Act’, which of the chapter 20, Law village courts in India nyayalayas With reference to Gram
following statements is/are and Justice established under Gram were setup Nyayalayas, consider the following
corect? http://www.downt Nyayalayas Act, 2008 for at many statements:
1. As per the Act, Gram oearth.org.in/cove speedy and easy access to locations 1. It has jurisdiction over both civil
Nyayalayas can hear only rage/where-are- justice system in the rural recently. and criminal cases.
civil cases and not criminal rural-courts-44754 areas. 2. The appeals against its decisions
cases. 1st statement is not correct. lie directly in High Courts.
2. The Act allows local social Gram Nyayalaya exercises the Which of the statements given
activists as powers of both Criminal and above is/are correct?
mediators/reconciliators. Civil Courts. (a) 1 only
Select the correct answer The Gram Nyayalaya are (b) 2 only
using the code given below. supposed to try to settle the (c) Both 1 and 2
(a) 1 only disputes as far as possible by (d) Neither 1 nor 2
(b) 2 only bringing about conciliation
(c) Both 1 and 2 between the parties and for
(d) Neither 1 nor 2 this purpose, it can make use
of the conciliators such social
activists/mediators.
66 Current With reference to the ‘Trans- D E CA http://www.thehin EN The Trans-Pacific Partnership Constantly Covered in PT365 material -
Affairs Pacific Partnership’, consider du.com/business/1 was signed by only 12 in news International Relations
the following statements: 2-nations-sign- member nations of Pacific Asked Twice in All India Test
1. It is an agreement among transpacific- rim, not all the Pacific Rim series Test- 1779, 2082, 1720
all the Pacific Rim countries partnership-trade- countries except China and The 12 Pacific Rim nations have
except China and Russia. deal/article819199 Russia. The members are: signed the TPP (Trans-Pacific
2. It is a strategic alliance for 2.ece Australia, Brunei, Canada, Partnership) Agreement which is
the purpose of maritime Chile, Japan, Malaysia, considered to be a game changer
security only. Mexico, New Zealand, Peru, for the global economy. Which of
Which of the statements Singapore, the United States the following could be the reasons
given above is/are correct? and Vietnam. for India having stayed out of the
(a) 1 only It is not a strategic alliance. It agreement?
(b) 2 only is a trade alliance. 1. Trade diversion
(c) Both 1 and 2 2. Loss of competitiveness of
(d) Neither 1 nor 2 Indian exports

50 www.visionias.in ©Vision IAS


3. Investment diversion
Select the correct answer using the
code given below.
(a) 1 and 2 only (b) 2 only
(c) 2 and 3 only (d) 1, 2 and 3
Test - 1724,1772,1891,1894,2070
Which of the following countries
is/are part of the Trans Pacific
Partnership Agreement?
1. Mexico
2. Australia
3. Vietnam
4. Japan
5. China
6. Indonesia
Select the correct answer using the
code given below.
(a) 1, 2, 3 and 4 only
(b) 2, 3 and 6 only
(c) 1, 4 and 5 only
(d) 1, 2, 3, 4, 5 and 6
67 Current Consider the following A M CA http://www.thehin EN The 3rd India-Africa summit was Covered in PT365 material -
Affairs statements: du.com/opinion/e was held in New Delhi in constantly in International Relations.
The India-Africa Summit ditorial/indiaafrica- October 2015. The previous news
1. held in 2015 was the third forum-summit- two gatherings were held in
such Summit reaching-out-to- 2008 in New Delhi and in 2011
2. was actually initiated by africa/article78238 in Addis Ababa.
Jawaharlal Nehru in 1951 07.ece
Which of the statements
given above is/are correct?
(a) 1 only
(b) 2 only
(c) Both 1 and 2
(d) Neither 1 nor 2
68 Economy What is/are the C M FCA http://www.thehin EN To ensure better transmission was Covered in PT365 material –
purpose/purposes of the du.com/business/E and transparency of rates, the constantly in Economics Covered in tests twice
‘Marginal Cost of Funds conomy/rbi-links- RBI introduced the MCLR news Test - 2057, 2139
based Lending Rate (MCLR)’ lending-rates-to- framework, under which Consider the following statements
announced by RBI? loan- banks have to calculate their regarding Marginal Cost of funds
1. These guidelines help tenor/article80010 cost of funds based on the based Lending rate (MCLR):
improve the transparency in 20.ece latest rates offered on 1. It refers to the minimum

51 www.visionias.in ©Vision IAS


the methodology followed https://rbi.org.in/S deposits or borrowings. This interest rate for loans that is
by banks for determining the CRIPTs/BS_PressRe was to ensure that changes in determined by a bank.
interest rates on advances. leaseDisplay.aspx? deposit rates were 2. It is not sensitive to changes in
2. These guidelines help prid=35749 immediately reflected on the policy rates thereby improving
ensure availability of bank banks’ cost of funds. monetary transmission.
credit at interest rates which Apart from helping improve Which of the statements given
are fair to the borrowers as the transmission of policy above is/are correct?
well as the banks. rates into the lending rates of (a) 1 only
Select the correct answer banks, these measures are (b) 2 only
using the code given below. expected to improve (c) Both 1 and 2
(a) 1 only transparency in the (d) Neither 1 nor 2
(b) 2 only methodology followed by Open test 4 – 2057
(c) Both 1 and 2 banks for determining interest Consider the following differences
(d) Neither 1 nor 2 rates on advances. The between Marginal Cost of Funds
guidelines are also expected based Lending Rate (MCLR) and
to ensure availability of bank Base Rate:
credit at interest rates which 1. Unlike base rate calculation,
are fair to the borrowers as Cash Reserve Ratio (CRR) cost is
well as the banks. considered in the calculation of
Further, marginal cost pricing MLCR.
of loans will help the banks 2. Unlike base rate calculation,
become more competitive Repo rate is mandatory in the
and enhance their long run calculation of MCLR.
value and contribution to Which of the statements given
economic growth. above is/are correct?
(a) 1 only
(b) 2 only
(c) Both 1 and 2
(d) Neither 1 nor 2
69 Environment What is/are unique about A D FCA http://www.thehin EN The Kharai camel grazes on was in news Covered in tests of 1717, 1768,
‘Kharai camel’, a breed found du.com/opinion/o mangroves and is tolerant to 2066
in India? p-ed/the-sinking- saline water. It is found only A dry land and coastal ecosystem
1. It is capable of swimming ship-of-the- in Gujarat’s coastal areas. It is is the natural habitat of an Indian
up to three kilometres in desert/article7671 capable of swimming up to animal species. The animal has no
seawater. 220.ece three kilometers into the sea predators in that area but its
2. It survives by grazing on in search of mangroves. existence is threatened due to the
mangroves. http://www.downt 3rd statement is incorrect. It destruction of its habitat. Which
3. It lives in the wild and oearth.org.in/cove can be domesticed. Given the one of the following could be that
cannot be domesticated. rage/the-sinking- breed’s ability to survive both animal?
Select the correct answer ship-40705 on land and sea, the Kharai (a) Saltwater crocodile
using the code given below. camel is one of the most (b) Dugong

52 www.visionias.in ©Vision IAS


(a) 1 and 2 only preferred choices of graziers (c) Kharai camel
(b) 3 only in the arid coastal region of (d) Wild water buffalo
(c) 1 and 3 only Kachchh. People consume its
(d) 1, 2 and 3 milk, while male calves are
sold for economic returns.
70 Current Recently, our scientists have A M CA http://www.thehin EN Scientists at the Botanical was in news
Affairs discovered a new and du.com/news/nati Survey of India (BSI)
distinct species of banana onal/other- discovered the new species of
plant which attains a height states/andamans- banana from a remote
of about 11 metres and has yield-a-sweet- tropical rain forest on the
orange-coloured fruit pulp. banana-with- Little Andaman islands.
In which part of India has it orange- The new species has orange-
been discovered? pulp/article775031 coloured fruit pulp and is
(a) Andaman Islands 6.ece about 11 metres high,
(b) Anaimalai Forests whereas as the usual banana
(c) Maikala Hills species is about three to four
(d) Tropical rain forests of metres high.
northeast
71 Current Which one of the following is C M CA http://www.thehin EN INS Astradharini is India’s first was in news Covered in PT365 material -
Affairs the best description of ‘INS du.com/todays- totally indigenously designed Science and Technology
Astradharini’, that was in the paper/tp- (95%) and built torpedo
news recently? national/ins- launch and recovery vessel
(a) Amphibious warfare ship astradharini- (TLRV).
(b) Nuclear-powered commissioned/arti
submarine cle7731983.ece
(c) Torpedo launch and
recovery vessel
(d) Nuclear-powered aircraft
carrier
72 Current What is ‘Greased Lightning- A D CA http://www.thehin EN The Greased Lightning or GL- recent
Affairs 10 (GL-10)’, recently in the du.com/sci- 10 is the electric plane with 10 developmen
news? tech/science/nasas engines tested by NASA that ts in science
(a) Electric plane tested by -10engine-electric- can take off and land like a and
NASA plane-completes- helicopter and fly efficiently technology
(b) Solar-powered two- flight- like an aircraft.
seater aircraft designed by test/article717046
Japan 8.ece
(c) Space observatory
launched by China
(d) Reusable rocket designed
by ISRO
53 www.visionias.in ©Vision IAS
54 www.visionias.in ©Vision IAS
73 Current With reference to ‘Initiative C D CA http://agricoop.nic RR The statement 1 is correct. initiatives of
Affairs for Nutritional Security .in/Admin_Agricoo The Initiative for Nutritional the
through Intestive Millets p/Uploaded_File/I Security through Intensive government
Promotion’, which of the NSIMP.pdf Millets Promotion aims to to promote
following statements is/are demonstrate the improved food
correct? production and post-harvest security
1. This initiative aims to technologies in an integrated
demonstrate the improved manner with cluster
production and post-harvest approach.
technologies, and to Statement 2 is also correct.
demonstrate value addition Millets are mainly grown in
techniques, in an integrated regions of low annual rainfall
manner, with cluster and poor arid soil. It is poor,
approach. small, marginal and tribal
2. Poor, small, marginal and farmers, who cannot afford
tribal farmers have larger costly irrigation practices,
stake in this scheme. depend on cultivation of
3. An important objective of millets. Hence these farmers
the scheme isto encourage benefit from the scheme and
farmers of commercial crops have a larger stake in this
to shift to millet cultivation scheme.
by offering them free kits of Statement 3 is not correct.
critical inputs of nutrients There is no such provision of
and microirrigation encouraging farmers of
equipment. commercial crops to shift to
Select the the correct answer millet cultivation.
using the code given below.
(a) 1 only
(b) 2 and 3 only
(c) 1 and 2 only
(d) 1, 2 and 3
74 Modern The ‘Swadeshi’ and ‘Boycott’ A E F NCERT 12 - EM The Swadeshi and Boycott to check Asked in Test – 1786
History were adopted as methods of Modern India by were adopted as methods of fundamenta Which of the following factors
struggle for the first time Bipin Chandra, struggle for the first time l/basic was/were responsible for the
during the page 195 during the agitation against knowledge decline of the Swadeshi
(a) agitation against the the Partition of Bengal in movement?
Partition of Bengal Swadeshi movement. 1. Repression by the government
(b) Home Rule Movement 2. Split in the Congress
(c) Non-Cooperation 3. Lack of effective organization
Movement 4. Inability of mass movement to
(d) visit of the Simon be sustained for long periods.

55 www.visionias.in ©Vision IAS


Commission to India Select the correct answer using the
code given below.
(a) 1, 2 and 3 only
(b) 1, 3 and 4 only
(c) 2 and 4 only
(d) 1, 2, 3 and 4
75 Culture With reference to the B D F Themes in Indian EM Bodhisattva in Buddhism is to check Asked in Open Test - 3 Test -1893
religious history of India, History - I (NCERT one who seeks awakening or basic/funda With reference to sects of
consider the following Class XII) enlightment. This concept is mental Buddhism, consider the following
statements: https://www.brita central to Mahayana sect of knowledge statements:
1. The concept of nnica.com/topic/b Buddhism. in Indian 1. The idea of transference of
Bodhisattva is central to odhisattva In Mahayana Buddhism, philosophy/ merit is integral to the teachings of
Hinayana sect of Buddhism. bodhisattva refers to a human culture Hinayana.
2. Bodhisattva is a being committed to the 2. Achievement of status of
compassionate one on his attainment of enlightenment 'Arhant' was integral to Mahayana.
way to enlightenment. for the sake of others. A 3. Vajrayana sect represents
3. Bodhisattva delays Bodhisattva may delay permeation of magic and
achieving his own salvation achieving his own salvation to mysticism in Buddhism.
to help all sentinent beings help all sentinent beings on Which of the statements given
on their path to it. their path to it. above is/are correct?
Which of the statements (a) 1 and 2 only
given above is/are correct? (b) 2 and 3 only
(a) 1 only (c) 3 only
(b) 2 and 3 only (d) 1, 2 and 3
(c) 2 only
(d) 1, 2 and 3
76 Current ‘Doctors Without Borders B E FCA http://www.thehin EN Médecins Sans Frontières was in news PT-365 material (May-June)
Affairs (Medecins Sans Frontieres)’, du.com/sci- (MSF) or Doctors without in relation to
often in the news, is tech/health/mdeci Borders, an international NGO RCEP
(a) a division of World Health ns-sans-frontires- has warned India that it will proposals;
Organization on-indias-role-in- no more remain ‘the also in news
(b) a non-governmental the-rcep- pharmacy of the developing when an US
international organization meet/article87286 world’ if the proposed military air
(c) an inter-governmental 09.ece proposals in the Regional strike hit a
hospital run
agency sponsored by Comprehensive Economic
by Doctors
European Union Partnership (RCEP) agreement
Without
(d) a specialized agency of are adopted.
Borders in
the United Nations Afghanistan
to oust
Talibani
insurgents
56 www.visionias.in ©Vision IAS
77 Environment With reference to an C D CAA http://www.thehin EN The initiative on 'The initiative to
initiative called ‘The du.com/books/liter Economics of Ecosystems and implement
Economics of Ecosystems ary-review/review- Biodiversity' (TEEB) is an CBD
and Biodiversity (TEEB)’, of-green-signals- important partner in programme/
which of the following ecology-growth- implementing the CBD climate
statements is/are correct? and-democracy-in- programme of work on change
1. It is an initiative hosted by india/article74075 incentive measures, and in
UNEP, IMF and World 56.ece particular its work on
Economic Forum. valuation. This major
2. It is a global initiative that http://www.thehin international initiative, funded
focuses on drawing attention dubusinessline.co by the European Commission,
to the economic benefits of m/opinion/how- Germany, United Kingdom,
biodiversity. green-is-my- Norway, the Netherlands and
3. It presents an approach balance- Sweden, and managed by the
that can help decision- sheet/article84474 United Nations Environment
makers recognize, 19.ece Programme as part of its
demonstrate and capture the Green Economy Initiative
value of ecosystems and (GEI), seeks to draw attention
biodiversity. to the global economic
Select the correct answer benefits of biodiversity, to
using the code given below. highlight the growing costs of
(a) 1 and 2 only biodiversity loss and
(b) 3 only ecosystem degradation, and
(c) 2 and 3 only to draw together expertise
(d) 1, 2 and 3 from the fields of science,
economics and policy to
enable practical actions
moving forward.
Members of the TEEB
Advisory Board include the
Executive Director of UNEP,
the Executive Secretary of the
CBD and the Director General
of IUCN, amongst others.
78 Environment With reference to ‘Red A M FCA http://www.thehin EN Red Sanders, botanical name was in news Tests - 1725, 1776
Sanders’, sometimes seen in du.com/news/nati Pterocarpus santalinus, is a in relation With reference to Red Sanders or
the news, consider the onal/andhra- non-fragrant variety of to smuggling Red Sandalwood, a tree species
following statements: pradesh/red- sandalwood that mostly of Red often in news, consider the
1. It is a tree species found in sanders-govt- grows in rocky, hilly regions. Sanders following statements:
a part of South India. steps-up-vigil-in- Saplings reach 8 to 10 m in 3- 1. It is an endangered species
2. It is one of the most seshachalam- 4 years, but growth slows found in the Palakonda and

57 www.visionias.in ©Vision IAS


important trees in the hills/article866171 down after that. The trunks Seshachalam hills of the Eastern
tropical rain forest areas of 8.ece are slender, and it takes at Ghats.
South India. least 20-25 years for the 2. It is highly valued in the
Which of the statements http://indianexpre tree’s beautiful, deep red international market for its rich
given above is/are correct? ss.com/article/expl wood to be of use. fragrance.
(a) 1 only ained/explained- It is found in the thorny 3. It is found extensively in the
(b) 2 only logs-of-blood/ scrub/dry deciduous forests of tropical wet evergreen forests of
(c) Both 1 and 2 the central Deccan, between India.
(d) Neither 1 nor 2 500 ft and 3000 ft. only in a Which of the statements given
small pocket roughly 5,200 sq above is/are correct?
km in the Palakonda and (a) 1 only
Seshachalam hills in the (b) 1 and 2 only
districts of Kadapa and (c) 2 and 3 only
Chittoor, in some contiguous (d) 1, 2 and 3
areas of Anantapur district, in
the Nallamalla forests in
Kurnool and Prakasam, and in
parts of Nellore district. Some
contiguous patches in Tamil
Nadu and Karnataka see some
wild growth.
Red Sanders is a protected
species under the Convention
on International Trade in
Endangered Species (CITES) of
Wild Fauna and Flora.
79 Environment Which of the following D M FA http://www.thehin EN Reducing Emissions from it was
statements is/are correct? du.com/news/inter Deforestation and Forest discussed in
Proper design and effective national/wonderin Degradation (REDD) attempts UNFCCC
implementation of UN- g-whats-cop21- to create financial value for summit in
REDD+ Programme can adp-heres-a- the carbon stored in forests, Paris(COP-
significantly contribute to glossary-for-un- offering incentives for 21)
1. protection of biodiversity climate- developing countries to
2. resilience of forest talks/article792994 reduce emissions from
ecosystems 8.ece forested lands and invest in
3. poverty reduction low-carbon paths to
Select the correct answer http://www.unep. sustainable development.
using the code given below. org/climatechange REDD+ goes beyond
(a) 1 and 2 only /reddplus/Introduc deforestation and forest
(b) 3 only tion/tabid/29525/ degradation, and includes the
(c) 2 and 3 only Default.aspx role of conservation,

58 www.visionias.in ©Vision IAS


(d) 1, 2 and 3 http://theredddesk sustainable management of
.org/what-redd forests and enhancement of
forest carbon stocks.
It is predicted that financial
flows for greenhouse gas
emission reductions from
REDD+ could reach up to
US$30 billion a year. This
significant north-south flow of
funds could reward a
meaningful reduction of
carbon emissions and support
new, pro-poor development,
help conserve biodiversity and
secure vital ecosystem
services.
80 Environment What is ‘Greenhouse Gas A D CA http://indianexpre EN The Greenhouse Gas (GHG) in news
Protocol’? ss.com/article/tech Protocol, developed by World because of
(a) It is an international nology/tech-news- Resources Institute (WRI) and reduction in
accounting tool for technology/apple- World Business Council on carbon
government and business iphone-ipad-how- Sustainable Development emission
leaders to understand, long-will-it-last/ (WBCSD), sets the global
quantify and manage standard for how to measure,
greenhouse gas emissions. manage, and report
(b) It is an initiative of the greenhouse gas emissions.
United Nations to offer Many companies and
financial incentives to organizations around the
developing countries to world are using GHG Protocol
reduce greenhouse gas standards and tools to
emissions and to adopt eco- manage their emissions and
friendly technologies become more efficient,
(c) It is an inter- resilient, and prosperous
governmental agreement organizations.
ratified by all the member
countries of the United
Nations to reduce
greenhouse gas emissions to
specified levels by the year
2022 (d) It is one of the
multilateral REDD+ initiatives
hosted by the World Bank

59 www.visionias.in ©Vision IAS


81 Economy With reference to ‘Financial C M FCA http://www.thehin EN The Financial Stability and FSCDC was Test 1779
Stability and Development dubusinessline.co Development Council (FSDC) constantly in With reference to the Financial
Council’, consider the m/economy/india- under the chairmanship of news in Sector, consider the following
following statements: must-be-prepared- union Finance Minister was relation to statements:
1. It is an organ of NITI to-deal-with- set up by the government as the meeting 1. Forwards Markets Commission
Aayog. brexit-global- the apex-level forum in held in July (FMC) is the independent regulator
2. It is headed by the Union uncertainty- December 2010. 2016 with for the commodities market.
Finance Minister. fsdc/article881214 respect to 2. Financial Stability and
3. It monitors 7.ece rising bad Development Council is chaired by
macroprudential supervision loans with the Union Finance Minister.
of the economy. http://pib.nic.in/ne banks and 3. Financial Sector Appellate
Which of the statements wsite/PrintRelease. imapct on Tribunal (FSAT) is proposed to be
given above is/are correct? aspx?relid=130256 India global set up as an appellate tribunal for
(a) 1 and 2 only financial all financial regulators.
(b) 3 only http://indianexpre threats.
(c) 2 and 3 only ss.com/article/busi Which of the statements given
(d) 1, 2 and 3 ness/banking-and- above is/are correct?
finance/india-alert- (a) 1 and 2 only
to-global-financial- (b) 1 and 3 only
threats-fsdc-arun- (c) 2 and 3 only
jaitley-raghuram- (d) 1, 2 and 3
rajan-2895484/
(July 2016)
82 Environment With reference to ‘Agenda A M FA http://www.ncert. EM Agenda 21 is a non-binding, Agenda 21 Test 1782
21’, sometimes seen in the nic.in/ncerts/l/jess voluntarily implemented was asked in Which of the following owe their
news, consider the following 101.pdf action plan of the United context of origin to the Rio Earth Summit,
statements: Nations with regard to news 1992?
1. It is a global action plan for http://www.thehin sustainable development. It is coming with 1. Agenda 21
sustainable development. du.com/opinion/co a product of the Earth Summit respect to 2. United Nations Environment
2. It originated in the World lumns/the- (UN Conference on Sustainable Programme (UNEP)
Summit on Sustainable unfinished-agenda- Environment and Developmen 3. United Nations Framework
Development held in of-the-21st- Development) held in Rio de t Goals. Convention on Climate Change
Johannesburg in 2002. century/article697 Janeiro, Brazil, in 1992. 4. Convention on Biological
Which of the statements 5352.ece Diversity
given above is/are correct? Select the correct answer using the
(a) 1 only http://www.un.org code given below.
(b) 2 only /geninfo/bp/enviro (a) 1, 3 and 4 only
(c) Both 1 and 2 .html (b) 1 and 4 only
(d) Neither 1 nor 2 (c) 2 and 3 only
(d) 1, 2, 3 and 4

60 www.visionias.in ©Vision IAS


61 www.visionias.in ©Vision IAS
83 Modern Satya Shodhak Samaj C E F Spectrum, Bipin EM In Poona, Jyotirao Phooley, to check the
History organized Chandra, Old Ncert though a Mali by caste and of basic/funda
(a) a movement for comparatively little education mental
upliftment of tribals in Bihar started the association called knowledge
(b) a temple-entry the Satyashodhak Samaj in in modern
movement in Gujarat 1873 for asserting the worth indian
(c) an anti-caste movement of man irrespective of caste. history
in Maharashtra He demanded representation
(d) a peasant movement in of all classes of the Hindus in
Punjab all the local bodies, in services
and institutions and also
established a primary school
for the so-called untouchables
in Poona.

84 Science and Which of the following D M F NCERT XIIth EM Viruses are too small to be Basic
Technology statements is/are correct? BIOLOGY seen by the naked eye. They biological
Viruses can infect can't multiply on their own, so science
1. bacteria they have to invade a 'host' question.
2. fungi cell and take over its
3. plants machinery in order to be able
Select the correct answer to make more virus
using the code given below. particles.Virus infects all of
(a) 1 and 2 only the given organisms (Bacteria,
(b) 3 only Fungi and Plant).
(c) 1 and 3 only Mycoviruses are viruses that
(d) 1, 2 and 3 infect fungi. The majority of
mycoviruses have double-
stranded RNA (dsRNA)
genomes.
Many different viruses can
infect plants. Certain crops
are well known to be affected
by virus diseases including
geraniums, roses, Easter lilies,
dahlias, gladiolus, and tulips.
The Virus that Infects
Bacteria. Coliphage T4 is a
virus that looks like an alien
landing pod. With its six legs,
the bacteriophage attaches to

62 www.visionias.in ©Vision IAS


the surface of the much larger
bacteria Escherichia coli (E.
coli). Once attached, the
bacteriophage injects DNA
into the bacterium.
85 Current The term ‘Base Erosion and B E CAA http://www.thehin EN The 2015 final reports of the was PT 365 Full Length Test - 2102
Affairs Profit Shifting’ is sometimes dubusinessline.co Organisation for Economic Co- constantly in Which of the following statements
seen in the news in the m/opinion/column operation and Development news regarding Base Erosion Profit
context of s/all-you-wanted- (OECD)-led project on Base Sharing Project (BEPS) are correct?
(a) mining operation by to-know-about- Erosion and Profit Shifting 1. It is an initiative spearheaded by
multinational companies in beps/article775399 (BEPS) — which refer to the FATF.
resource-rich but backward 2.ece erosion of a nation’s tax base 2. It aims to stop shifting of
areas due to the accounting tricks of corporate profits to low tax
(b) curbing of the tax evasion http://www.thehin Multinational Enterprises countries from where the
by multinational companies du.com/opinion/le (MNEs) and the legal but economic activity takes place.
(c) exploitation of genetic ad/black-money- abusive shifting out of profits Choose the correct option using
resources of a country by the-hidden-wealth- to low-tax jurisdictions the code given below.
multinational companies of- respectively — lays out 15 (a) 1 only
(d) lack of consideration of nations/article813 action points to curb abusive (b) 2 only
environmental costs in the 0657.ece tax avoidance by MNEs. As a (c) Both 1 and 2
planning and participant of this project, (d) Neither 1 nor 2
implementation of http://pib.nic.in/ne India is expected to
developmental projects. wsite/PrintRelease. implement at least some of
aspx?relid=138550 these measures.
86 Current Recently, India’s first A M CA http://www.thehin EN Andhra Pradesh is set to house was in news
Affairs ‘National Investment and du.com/news/nati India's first national investment in relation
Manufacturing Zone’ was onal/andhra- and manufacturing zone after to protests
proposed to be set up in pradesh/nimz- the state assured the Centre of by the local
(a) Andhra Pradesh works-gain-pace- availability of 10 sq km of land population
(b) Gujarat in-prakasam- in one place in Prakasham demanding
(c) Maharashtra district/article8030 district. adequate
(d) Uttar Pradesh 363.ece The imminent final approval for compensati
the NIMZ, which is expected to
on for their
give a fillip to Make in India
http://www.newin land being
campaign, comes four years
dianexpress.com/s taken up by
after the concept was mooted
tates/telangana/Pr to boost manufacturing in the
the
otest-against-land- country and two years after the government
acquisition- Department of Industrial Policy to set up
spreads/2016/06/0 and Promotion gave an in- NIMZ
7/article3470099.e principle nod to Andhra
ce Pradesh in this regard.
63 www.visionias.in ©Vision IAS
87 Economy What is/are the B M FCA http://www.thehin EN District Mineral Foundation Setting up of Tests - 1723, 1774
purpose/purposes of ‘District dubusinessline.co (DMF) is a trust set up as a District With reference to the District
Mineral Foundations’ in m/economy/policy non-profit body, in those Mineral Mineral Foundation (DMF)
India? /govt-notifies- districts affected by the Foundations established under the Mines and
1. Promoting mineral miners- mining works, to work for the (DMFs) Minerals (Development and
exploration activities in contribution-to- interest and benefit of through the Regulation) Amendment Act, 2015,
mineral-rich districts district-mineral- persons and areas affected by Mines and consider the following statements:
2. Protecting the interests of foundation/article mining related operations. It Minerals 1. It will be funded from the
the persons affected by 7662554.ece is funded through the (Developme consolidated fund of the state.
mining operations contributions from miners. nt & 2. The fund will be used for
3. Authorizing State http://www.prsind Its manner of operation Regulation) developing the mining fields.
Governments to issue ia.org/billtrack/the comes under the jurisdiction Amendment 3. It will be set up in all districts
licenses for mineral -mines-and- of the relevant State Act, affected by mining activities.
exploration minerals- Government. (MMDRA) Which of the statements given
Select the correct answer development-and- 2015. On 16 above is/are correct?
using the code given below. regulation- September (a) 1 and 2 only
(a) 1 and 2 only amendment-bill- 2015, (b) 3 only
(b) 2 only 2015-3648/ Central (c) 1, 2 and 3
(c) 1 and 3 only Government (d) None
(d) 1, 2 and 3 issued a
notification
directing
states to set
up DMF.
88 Current ‘SWAYAM’, an initiative of D M CAA http://pib.nic.in/ne EN Study Webs of Active- In news, Tests – 1775
Affairs the Government of India, wsite/PrintRelease. Learning for Young Aspiring because of Which of the following best
aims at aspx?relid=133798 Minds (SWAYAM), is a Web Massive describes SWAYAM, an initiative of
(a) promoting the Self Help portal where Massive Open Open Online Ministry of Human Resouce
Groups in rural areas http://www.thehin On-line Courses (MOOCs) will Courses Development?
(b) providing financial and du.com/features/e be available on all kinds of (MOOC) are (a) A web portal where Massive
technical assistance to young ducation/mooc- subjects. SWAYAM is the fast gaining Open Online Courses will be
start-up entrepreneurs advantage/article8 Indian electronic e-education popularity available.
(c) promoting the education 144936.ece platform which proposes to among (b) A web portal for self
and health of adolescent girls offer courses from the high students certification of Online Vocational
(d) providing affordable and school stage to Post-Graduate and working Courses.
quality education to the stage in an interactive professional (c) An online portal for
citizens for free electronic platform. The IT s as they development of women and girl
platform for SWAYAM is going help them education programme.
to be built and is expected to study at (d) An online platform for foreign
be operationalized by 31st their universities to collaborate with
March 2016 with a capacity to convenience Indian counterparts.

64 www.visionias.in ©Vision IAS


host nearly 2,000 courses. and time.
Websites
such as edX
and
Coursera
enlist free
online
courses
from various
institutes
across the
world.
89 Modern The Montague-Chelmsford D E F NCERT XIIth std- EM On 20th August, 1917 Lord to test the Tests - 1721, 1772, 1894, 2062
History Proposals were related to Bipan Chandra Montague, the Secretary of fundamenta With reference to provisions/
(a) social reforms State for India, made the l knowledge outcomes of Montague
(b) educational reforms following declaration in British in modern Declaration, 1917, consider the
(c) reforms in police Parliament: “ The Policy of His Indian following statements:
administration Majesty’s government… is history 1. It provided for the
(d) constitutional reforms that of increasing association establishment of a responsible
of Indians in every branch of government in India.
administration, and the 2. The demand for self-
gradual development of self- government could not be termed
governing institutions, with a as seditious.
view to the progressive 3. It aimed at increasing
realization of responsible association of Indians in every
government in India as an branch of administration.
integral part of the British Which of the statements given
empire. above is/are correct?
Importance of Montagu's (a) 1 and 2 only
Statement- From now (b) 1 and 3 only
onwards, the demand by (c) 2 and 3 only
nationalists for self- (d) 1, 2 and 3
government or Home Rule
could not be termed as
seditious since attainment of
self-government for Indians
now became a government
policy, unlike Morley's
statement in 1909 that the
reforms were not intended to
give self-government to India.

65 www.visionias.in ©Vision IAS


90 Culture What is/are common to the B D FA NCERT- EM The Ajanta Caves in to test the
two historical places known Introduction to Maharashtra, a World fundamenta
as Ajanta and Indian Art Part-I Heritage Site, are 30 rock-cut l cultural
Mahabalipuram? cave Buddhist temples carved history of
1. Both were built in the into the sheer vertical side of India
same period. a gorge near a waterfall-fed
2. Both belong to the same pool located in the hills of the
religious denomination. Sahyadri mountains.
3. Both have rock-cut Guntapalle is a rock-cut cave
monuments. site near Eluru. The other
Select the correct answer important site where rock-cut
using the code given below. stupas have been excavated is
(a) 1 and 2 only Anakapalle near
(b) 3 only Vishakhapatanam.
(c) 1 and 3 only The Pallavas were one of the
(d) None of the statements ancient South Indian
given above is correct dynasties that were active in
the Andhra region from the
second century CE onwards
and moved south to settle in
Tamil Nadu. Their early
buildings, it is generally
assumed, were rockcut, while
the later ones were structural.
The tradition of rock-cut cave
continued in the Deccan and
they are found not only in
Maharashtra but also in
Karnataka, mainly at Badami
and Aihole, executed under
the patronage of the
Chalukyas; in Andhra Pradesh
in the area of Vijayawada; and
in Tamil Nadu, mainly at
Mahabalipuram, under the
patronage of the Pallavas.
91 Economy With reference to ‘Bitcoins’, B D CAA http://www.thehin EN Bitcoin is a digital currency was in news Tests - 1786, 1789 (Open Test)
sometimes seen in the news, du.com/business/a that is not tied to a bank or constantly With reference to Bitcoins,
which of the following ll-you-need-to- government and allows users consider the following statements:
statements is/are correct? know-about- to spend money 1. It is a digital code mined after
1. Bitcoins are tracked by the bitcoin/article8547 anonymously. The coins are solving a complex maths problem.

66 www.visionias.in ©Vision IAS


Central Banks of the 440.ece created by users who ''mine'' 2. It is used as peer to peer
countries. them by lending computing transaction without any central
2. Anyone with a Bitcoin http://www.financi power to verifying other bank.
address can send and receive alexpress.com/pho users' transactions. They 3. RBI has recognized and allowed
Bitcoins from anyone else tos/business- receive bitcoins in exchange. Bitcoins for daily transactions.
with a Bitcoin address. gallery/247548/bit The coins also can be bought Which of the statements given
3. Online payments can be coin-meaning- and sold on exchanges with above is/are correct?
sent without either side bitcoin-price-a- U.S. dollars and other (a) 1 only
knowing the identity of the look-at-the-digital- currencies. (b) 1 and 2 only
other. currency-bitcoin- Bitcoins have become popular (c) 3 only
Select the correct answer economy/ because transactions can be (d) 1 and 3 only
using the code given below. made anonymously, making
(a) 1 and 2 only the currency popular with With reference to Bitcoins,
(b) 2 and 3 only libertarians as well as tech consider the following statements:
(c) 3 only enthusiasts, speculators - and 1. It is a digitial currency, created
(d) 1, 2 and 3 criminals. and held electronically.
2. It is mined using special
softwares to solve mathematical
problems.
3. BitMEX is the sole exchange for
bitcoin trading.
4. It is not tradeable at the spot
market.
Which of the statements given
above is/are correct?
(a) 1 and 2 only (b) 2, 3 and 4 only
(c) 1, 3 and 4 only (d) 1, 2, 3 and 4
92 Current Consider the following B M CA http://www.thehin EN The New Development Bank NDB was in Tests - 1715,1766, 1890, 2060
Affairs statements: du.com/news/inter (NDB) BRICS group of nations news Consider the following statements
1. New Development Bank national/after- comprising Brazil, Russia, constantly with reference to " New
has been set up by APEC. forming-new- India, China and South Africa . Development Bank BRICS".
2. The headquarters of New development- To begin with, the bank will 1. The first President of NDB is an
Development Bank is in bank-brics-can- start operating with $50 Indian.
Shanghai. leadthe-global- billion in initial capital with 2. It will be headquartered in
Which of the statements south-chinese- the five BRICS contributing Shanghai, China.
given above is/are correct? scholar/article8886 $10 billion each. According to 3. In the New Development Bank
(a) 1 only 978.ece the pact, the capital of the each participant country will be
(b) 2 only bank will be divided equally assigned vote on the basis of
(c) Both 1 and 2 among the five participating capital share.
(d) Neither 1 nor 2 nations and initially it will 4. The Bank shall mobilize
focus on infrastructure resources for infrastructure and

67 www.visionias.in ©Vision IAS


projects member countries. sustainable development projects
The headquarters of the bank in BRICS as well as other countries.
will be located in Shanghai, Which of the statements given
China. above is/are correct?
Eminent banker Kundapur (a) 1 and 2 only
Vaman Kamath was appointed (b) 1, 2 and 3 only
as first President New (c) 1, 2 and 4 only
Development Bank (NDB) of (d) 2, 3 and 4 only
BRICS nations.
The Bank shall mobilize
resources for infrastructure
and sustainable development
projects in BRICS and other
emerging economies and
developing countries,
complementing the existing
efforts of multilateral and
regional financial institutions
for global growth and
development
Unlike the World Bank, which
assigns votes based on capital
share, in the New
Development Bank each
participant country will be
assigned one vote, and none
of the countries will have veto
power.
93 Environment ‘Gadgil Committee Report’ D E FCA http://www.thehin EN The Kasturirangan panel was Kasturiranga PT 365 Environment material - Pg
and ‘Kasturirangan du.com/news/nati set up to study the Gadgil n panel was 31
Committee Report’, onal/kerala/fightin committee report on the in news
sometimes seen in the news, g-over-the- Western Ghats. The
are related to western- Kasturirangan report seeks to
(a) constitutional reforms ghats/article74902 bring just 37% of the Western
(b) Ganga Action Plan 93.ece Ghats under the Ecologically
(c) linking of rivers Sensitive Area (ESA) zones —
(d) protection of Western down from the 64% suggested
Ghats by the Gadgil report.
Recommended prohibition on
development and commercial
activities like mining, thermal

68 www.visionias.in ©Vision IAS


power plants, polluting
industries and large housing
plans in Ecologically Sensitive
Area (ESA) zones.
94 Modern Consider the following: B D F http://www.thebra RR Raja Ram Mohan Roy along
History 1. Calcutta Unitarian hmosamaj.net/fou with Dwarka Nath Tagore and
Committee nders/keshub.html William Adam established
2. Tabernacle of New Calcutta Unitarian committee
Dispensation and 1823. Keshubh Chandra
3. Indian Reform Association was not associated with it.
Keshab Chandra Sen is On 24th January 1868, Keshub
associated with the laid the foundation stone of
establishment of which of his new church, the Brahmo
the above? Samaj of India Tabernacle of
(a) 1 and 3 only New Dispensation and the
(b) 2 and 3 only newly constructed chapel was
(c) 3 only (d) 1, 2 and 3 consecrated on 22nd August
1869.
95 Current Which of the following is not A E FCA http://www.thehin EN The Cooperation Council for It was in Tests- 1715, 1766, 1890
Affairs a member of ‘Gulf du.com/news/nati the Arab States of the Gulf, news Which of the following countries
Cooperation Council’? onal/swaraj- originally known as the Gulf constantly are the members of Cooperation
(a) Iran pushes-for- Cooperation Council (GCC), is due to Council for the Arab States of the
(b) Saudi Arabia indiagulf- a regional inter governmental Indian Prime Gulf?
(c) Oman cooperation- political and economic union Minister's 1. Bahrain
(d) Kuwait council-free-trade- consisting of all Arab states of visit to UAE 2. Oman
agreement/article7 the Persian Gulf, except for and Saudi 3. Kuwait
710985.ece Iraq. Its member states are Arabia to 4. Iraq
Bahrain, Kuwait, Oman, Qatar, give a spur 5. Libya
Saudi Arabia, and the United to India and Select the correct answer using the
Arab Emirates. GCC code given below.
countries; (a) 1, 2 and 3 only
US President (b) 1, 3 and 4 only
attended (c) 2, 3, 4 and 5 only
GCC (d) 1, 2, 4 and 5 only
Summit.
96 Economy What is/are the C M CAA http://www.thehin EN The government had was in news Tests - 1713, 1765
purpose/purposes of du.com/opinion/co launched three ambitious constantly Which of the following schemes
Government’s ‘Sovereign lumns/the-sum-of- schemes to reduce the targets to reduce the import for
Gold Bond Scheme’ and three-new-gold- physical demand for gold and gold?
‘Gold Monetization schemes/article78 fish out 20,000 tonnes of the 1. Sovereign gold bonds
Scheme’? 69915.ece precious metal worth $800 2. Gold Monetisation
69 www.visionias.in ©Vision IAS
1. To bring the idle gold lying billion lying idle with 3. Indian Gold Coin scheme
with Indian households into households. PM launched the Select the correct answer using the
the economy. maiden sovereign gold bond, code given below.
2. To promote FDI in the gold gold monetisation and the (a) 1 only
and jewellery sector Indian gold coin scheme. The (b) 2 and 3 only
3. To reduce India’s main objectives of the (c) 1, 2 and 3
dependence on gold imports schemes is to reduce India's (d) None of the above
Select the correct answer gold imports and bring all the
using the code given below. gold lying idle with individuals
(a) 1 only (b) 2 and 3 only and households.
(c) 1 and 3 only (d) 1, 2 and 3
97 Current ‘Belt and Road Initiative’ is D E FCA http://www.thehin EN It is a developmental strategy was Test code: 1893, PT-365(
Affairs sometimes mentioned in the du.com/opinion/o and framework, unveiled by constantly in International Relations, pg 37)
news in the context of the p-ed/chinas-one- Chinese leader Xi Jinping in news With reference to 'One Belt One
affairs of belt-one-road- September and October 2013 Road' (OBOR) initiative, consider
(a) African Union programme/article in announcements revealing the following statements:
(b) Brazil 8179870.ece the SREB (Silk Road Economic 1. It aims to connect Asia, Europe
(c) European Union Belt) and MSR (Maritime Silk and Africa.
(d) China Road), respectively. 2. Both maritime route and road
The “belt and road” run route are an integral part of the
through the continents of initiative.
Asia, Europe and Africa. 3. China-Pakistan economic
It focuses on connectivity and corridor is a part of OBOR.
cooperation among countries Which of the statements given
primarily in Eurasia, which above is/are correct?
consists of two main (a) 1 only
components, the land-based (b) 1 and 2 only
SREB and oceangoing MSR (c) 2 and 3 only
and it is known as “One Belt (d) 1, 2 and 3
One Road”(OBOR).
The “belt and road” would be
serviced by a network of
roads, high-speed railways,
fibre-optic lines,
transcontinental submarine
optical cable projects, and
satellite information
passageways.

70 www.visionias.in ©Vision IAS


71 www.visionias.in ©Vision IAS
98 Current Pradhan Mantri MUDRA A E CAA http://www.thehin EN Pradhan Mantri MUDRA was in news Test code: 1723, 1774
Affairs Yojana is aimed at du.com/business/c Yojana is aimed at bringing constantly With reference to MUDRA (Micro
(a) bringing the small abinet-approves- sole-proprietors or Units Development and Refinance
entrepreneurs into formal conversion-of- entrepreneurs of Small & Agency) Bank, consider the following
financial system mudra-into- Medium Enterprises into statements:
(b) providing loans to poor bank/article80728 formal financial system. 1. The funding to small
farmers for cultivating 87.ece entrepreneurs will be disbursed
particular crops through both banks and non-banking
financial institutions (NBFCs).
(c) providing pensions to old
2. The bank will provide more than
and destitute persons
50% of the loans to SC/ST
(d) funding the voluntary
entrepreneurs.
organizations involved in the
3. MUDRA Bank is both financier and
promotion of skill regulator of Micro Finance
development and Institutions (MFIs).
employment generation Which of the statements given
above is/are true?
(a) 1 only (b) 1 and 3 only
(c) 2 only (d) 1, 2 and 3
99 Geography In which of the following D D FCA http://pib.nic.in/ne EN Shale gas is the natural gas It was in
regions of India are shale gas wsite/PrintRelease. that is trapped within shale news
resources found? aspx?relid=121651 formations. ONGC sought because this
1. Cambay Basin permission for drilling 11 is the first
2. Cauvery Basin http://www.thehin exploratory wells for shale oil time that
3. Krishna-Godavari Basin du.com/todays- and shale gas in Cambay basin the ONGC
Select the correct answer paper/tp- at Mehsana, Ahmedabad and has taken up
using the code given below. business/ongc-to- Bharuch districts of Gujarat, shale gas
(a) 1 and 2 only drill-17- one well in Cauvery basin at exploration
(b) 3 only exploratory-wells- Nagapattinam in Tamil Nadu in such a big
(c) 2 and 3 only for-shale-gas- and five wells in KG Basin at scale. Also,
(d) 1, 2 and 3 oil/article8517524. East and West Godavari it first time
ece districts of Andhra Pradesh. that it has
Other regions in India where taken up
shale gas resources are found shale gas
- Cauvery, Ganga & Assam and exploration
Assam - Arakan; Gondwana in the
basin (including Damodar). Krishna-
Godavari
basin.

72 www.visionias.in ©Vision IAS


100 Current ‘Global Financial Stability B E FCA http://www.thehin EN Global Financial Stability was in news Tests - 2057 (Open Test)
Affairs Report’ is prepared by the du.com/business/E Report is published by constantly Which among the following
(a) European Central Bank conomy/world- International Monetary Fund. organizations releases Global
(b) International Monetary output-faces-risk- The financial stability report Financial Stability Report?
Fund of-39-drop-by- assesses the risks faced by the (a) World Bank
(c) International Bank for 2021/article84722 global financial system. (b) United Nations Development
Reconstruction and 98.ece The latest report released, Programme
Development warns that global output (c) International Monetary Fund
(d) Organization for could decline 3.9 per cent by (d) International Labour
Economic Cooperation and 2021 if action is not taken to Organisation
Development address the risks faced by the
financial system. The main
message of this report is that
additional measures are
needed to deliver a more
balanced and potent policy
mix for improving the growth
and inflation outlook and
securing financial stability.

73 www.visionias.in ©Vision IAS


TOPIC

Number of Questions by
Topic Number of Questions by Topic
Topic
Modern History 6
Medieval History 4
Ancient History 3 6%
4%
Culture 3 3%
Modern History
Science and 3%
Medieval History
Technology 7 37% Ancient History
7%
Economy 15 Culture

Geography 3 Science and Technology


Economy
Polity and
Geography
Governance 6 15%
Polity and Governance
Environment 16 Environment

Current Affairs 37 Current Affairs


3%
Grand Total 100
16% 6%

74 www.visionias.in ©Vision IAS


DIFFICULTY

Level Number of Questions by Difficulty Number of Questions by Difficulty


Easy 27

Medium 50
23%
Difficult 23 27%

Easy
Medium
Difficult

50%

75 www.visionias.in ©Vision IAS


NATURE

Nature Number of Questions by Nature


Number of Questions by Nature
CA 27

CAA 20
20%
F 19 27%

FA 14 CA
CAA
FCA 20 F
14%
FA
FCA

20%

19%

76 www.visionias.in ©Vision IAS


SECTIONWISE NATURE
Section wise Nature CA CAA F FA FCA Grand Total
Culture 2 1 3
Current Affairs 22 8 1 6 37
Economy 1 3 4 7 15
Environment 2 4 1 3 6 16
Geography 1 1 1 3
Ancient History 2 1 3
Medieval History 4 4
Modern History 6 6
Polity and Governance 2 3 1 6
Science and Technology 2 1 4 7
Grand Total 100

Culture, Current Affairs, Economy, Environment, Geography…


30

science and technology


25
Polity and Governance
20 Modern History
Medieval History
15
Ancient History
10 Geography
Environment
5
Economy

0 Current Affairs
CA CAA F FA FCA Culture
Section wise Nature

77 www.visionias.in ©Vision IAS


SECTIONWISE DIFFICULTY
Section wise Difficulty Easy Medium Difficult Grand Total
Culture 3 3
Current Affairs 14 19 4 37
Economy 3 11 1 15
Environment 3 9 4 16
Geography 1 2 3
Ancient History 1 2 3
Medieval History 1 3 4
Modern History 5 1 6
Polity 1 4 1 6
Science and Technology 1 4 2 7
Grand Total 100

Culture, Current Affairs, Economy, Environment, Geography…


60

50 science and technology


Polity
40 Modern History
Medieval History
30
Ancient History

20 Geography
Environment
10 Economy
Current Affairs
0
Culture
Easy Medium Difficult
Section wise Difficulty

78 www.visionias.in ©Vision IAS


SOURCE TYPE

Number of Questions by Source


Type
Source Type Number of Questions by Source Type

EM 24 10%
24%
EN 66
EM
RR 10 EN
RR
Grand Total 100
66%

79 www.visionias.in ©Vision IAS


80 www.visionias.in ©Vision IAS

You might also like